A Literally Massive Problem

Article Type
Changed
Thu, 05/09/2019 - 00:01
Display Headline
A Literally Massive Problem

A 60-year-old woman presents to dermatology with a longstanding complaint of a tender, irritated mass hanging from her lower abdomen. The patient says it started as a large fold in her lower abdomen but over the years has grown and become more pendulous. It is now large enough to interfere with normal activity, including walking.

Numerous providers, dermatologists included, have rendered diagnoses—most recently, hidradenitis suppurativa. The antibiotics prescribed for that diagnosis have not helped, however. Similarly, cultures performed on samples from draining sores on the mass’s posterior have failed to illuminate the situation, showing only mixed normal flora.

The patient’s primary care provider referred her to surgery for consideration of removal, or at least reduction, of the mass. The surgeon offered a presumptive diagnosis of elephantiasis nostras verrucosa of the pannus and agreed to perform the surgery. But the patient’s primary care provider requested a second opinion from dermatology.

A Literally Massive Problem

EXAMINATION
The edematous, pendulous mass is the size of a soccer ball and hangs down from its abdominal base. When the patient stands, the mass stretches almost to the level of her knees. The anterior surface is edematous but otherwise normal in appearance. The intertriginous surfaces of the lesion look entirely different, with multiple small, draining puncta and a few open comedones.

A Literally Massive Problem

The body of the mass is quite indurated but is neither hot nor tender. There are no comedones or cysts in other intertriginous locations, as might be seen in hidradenitis.

What’s the diagnosis?

 

 

DISCUSSION
This case involves a rare entity: vast lymphedema of the pannus leading to the formation of a pendulous mass so large that it filled the space between the patient’s legs, causing pain and discomfort. These findings are analogous to those seen in advanced venous insufficiency. Both manifestations share a name: elephantiasis nostras verrucosa. (Neither has anything to do with the more notorious cases of filarial elephantiasis seen in tropical locations.)

Elephantiasis nostras verrucosa of the lower extremities involves striking skin changes: edema, along with extreme thickening, papularity, and roughness of the skin. These typically manifest downward from just below the knee. The condition represents the effects of late-stage chronic venous insufficiency, often worsened by obesity and a sedentary lifestyle. Other causes of dependent lymphedema, such as congestive heart failure, can also contribute to the problem.

This same pathophysiologic process can affect other areas as well—including the pannus, as seen in this case. Since I had only ever encountered this problem in legs, I did a literature search. I found several references, all of which indicated that surgical removal (panniculectomy) was the best treatment. I could not find any information on the success rate of this surgery, but I did refer the patient back to the surgeon, who had made the correct diagnosis.

TAKE-HOME LEARNING POINTS

  • Elephantiasis nostras verrucosa (ENV) is a consequence of uncontrolled venous insufficiency that commonly manifests on lower extremities.
  • ENV is a distinctly rare (though not unknown) problem when areas other than legs are affected.
  • This patient’s condition is, in my opinion, beyond the reach of medical treatment. But in milder cases, approaches such as weight loss and use of diuretics have been tried with mixed success.
  • The best treatment appears to be surgical removal, which is not without potential complications: risk for infection, pain, prolonged recovery time, and wound dehiscence; these issues were discussed thoroughly with the case patient.
Publications
Topics
Sections

A 60-year-old woman presents to dermatology with a longstanding complaint of a tender, irritated mass hanging from her lower abdomen. The patient says it started as a large fold in her lower abdomen but over the years has grown and become more pendulous. It is now large enough to interfere with normal activity, including walking.

Numerous providers, dermatologists included, have rendered diagnoses—most recently, hidradenitis suppurativa. The antibiotics prescribed for that diagnosis have not helped, however. Similarly, cultures performed on samples from draining sores on the mass’s posterior have failed to illuminate the situation, showing only mixed normal flora.

The patient’s primary care provider referred her to surgery for consideration of removal, or at least reduction, of the mass. The surgeon offered a presumptive diagnosis of elephantiasis nostras verrucosa of the pannus and agreed to perform the surgery. But the patient’s primary care provider requested a second opinion from dermatology.

A Literally Massive Problem

EXAMINATION
The edematous, pendulous mass is the size of a soccer ball and hangs down from its abdominal base. When the patient stands, the mass stretches almost to the level of her knees. The anterior surface is edematous but otherwise normal in appearance. The intertriginous surfaces of the lesion look entirely different, with multiple small, draining puncta and a few open comedones.

A Literally Massive Problem

The body of the mass is quite indurated but is neither hot nor tender. There are no comedones or cysts in other intertriginous locations, as might be seen in hidradenitis.

What’s the diagnosis?

 

 

DISCUSSION
This case involves a rare entity: vast lymphedema of the pannus leading to the formation of a pendulous mass so large that it filled the space between the patient’s legs, causing pain and discomfort. These findings are analogous to those seen in advanced venous insufficiency. Both manifestations share a name: elephantiasis nostras verrucosa. (Neither has anything to do with the more notorious cases of filarial elephantiasis seen in tropical locations.)

Elephantiasis nostras verrucosa of the lower extremities involves striking skin changes: edema, along with extreme thickening, papularity, and roughness of the skin. These typically manifest downward from just below the knee. The condition represents the effects of late-stage chronic venous insufficiency, often worsened by obesity and a sedentary lifestyle. Other causes of dependent lymphedema, such as congestive heart failure, can also contribute to the problem.

This same pathophysiologic process can affect other areas as well—including the pannus, as seen in this case. Since I had only ever encountered this problem in legs, I did a literature search. I found several references, all of which indicated that surgical removal (panniculectomy) was the best treatment. I could not find any information on the success rate of this surgery, but I did refer the patient back to the surgeon, who had made the correct diagnosis.

TAKE-HOME LEARNING POINTS

  • Elephantiasis nostras verrucosa (ENV) is a consequence of uncontrolled venous insufficiency that commonly manifests on lower extremities.
  • ENV is a distinctly rare (though not unknown) problem when areas other than legs are affected.
  • This patient’s condition is, in my opinion, beyond the reach of medical treatment. But in milder cases, approaches such as weight loss and use of diuretics have been tried with mixed success.
  • The best treatment appears to be surgical removal, which is not without potential complications: risk for infection, pain, prolonged recovery time, and wound dehiscence; these issues were discussed thoroughly with the case patient.

A 60-year-old woman presents to dermatology with a longstanding complaint of a tender, irritated mass hanging from her lower abdomen. The patient says it started as a large fold in her lower abdomen but over the years has grown and become more pendulous. It is now large enough to interfere with normal activity, including walking.

Numerous providers, dermatologists included, have rendered diagnoses—most recently, hidradenitis suppurativa. The antibiotics prescribed for that diagnosis have not helped, however. Similarly, cultures performed on samples from draining sores on the mass’s posterior have failed to illuminate the situation, showing only mixed normal flora.

The patient’s primary care provider referred her to surgery for consideration of removal, or at least reduction, of the mass. The surgeon offered a presumptive diagnosis of elephantiasis nostras verrucosa of the pannus and agreed to perform the surgery. But the patient’s primary care provider requested a second opinion from dermatology.

A Literally Massive Problem

EXAMINATION
The edematous, pendulous mass is the size of a soccer ball and hangs down from its abdominal base. When the patient stands, the mass stretches almost to the level of her knees. The anterior surface is edematous but otherwise normal in appearance. The intertriginous surfaces of the lesion look entirely different, with multiple small, draining puncta and a few open comedones.

A Literally Massive Problem

The body of the mass is quite indurated but is neither hot nor tender. There are no comedones or cysts in other intertriginous locations, as might be seen in hidradenitis.

What’s the diagnosis?

 

 

DISCUSSION
This case involves a rare entity: vast lymphedema of the pannus leading to the formation of a pendulous mass so large that it filled the space between the patient’s legs, causing pain and discomfort. These findings are analogous to those seen in advanced venous insufficiency. Both manifestations share a name: elephantiasis nostras verrucosa. (Neither has anything to do with the more notorious cases of filarial elephantiasis seen in tropical locations.)

Elephantiasis nostras verrucosa of the lower extremities involves striking skin changes: edema, along with extreme thickening, papularity, and roughness of the skin. These typically manifest downward from just below the knee. The condition represents the effects of late-stage chronic venous insufficiency, often worsened by obesity and a sedentary lifestyle. Other causes of dependent lymphedema, such as congestive heart failure, can also contribute to the problem.

This same pathophysiologic process can affect other areas as well—including the pannus, as seen in this case. Since I had only ever encountered this problem in legs, I did a literature search. I found several references, all of which indicated that surgical removal (panniculectomy) was the best treatment. I could not find any information on the success rate of this surgery, but I did refer the patient back to the surgeon, who had made the correct diagnosis.

TAKE-HOME LEARNING POINTS

  • Elephantiasis nostras verrucosa (ENV) is a consequence of uncontrolled venous insufficiency that commonly manifests on lower extremities.
  • ENV is a distinctly rare (though not unknown) problem when areas other than legs are affected.
  • This patient’s condition is, in my opinion, beyond the reach of medical treatment. But in milder cases, approaches such as weight loss and use of diuretics have been tried with mixed success.
  • The best treatment appears to be surgical removal, which is not without potential complications: risk for infection, pain, prolonged recovery time, and wound dehiscence; these issues were discussed thoroughly with the case patient.
Publications
Publications
Topics
Article Type
Display Headline
A Literally Massive Problem
Display Headline
A Literally Massive Problem
Sections
Disallow All Ads
Content Gating
No Gating (article Unlocked/Free)
Alternative CME
Disqus Comments
Default
Gate On Date
Wed, 05/08/2019 - 12:45
Un-Gate On Date
Wed, 05/08/2019 - 12:45
Use ProPublica
CFC Schedule Remove Status
Wed, 05/08/2019 - 12:45
Hide sidebar & use full width
render the right sidebar.

Was Declining Treatment a Bad Idea?

Article Type
Changed
Thu, 04/11/2019 - 00:01
Display Headline
Was Declining Treatment a Bad Idea?

A 50-year-old African-American man is referred to dermatology by his primary care provider for evaluation of colored stripes in most of his fingernails. These have been present, without change, for most of his adult life.

The patient has been told these changes probably represent fungal infection, but being dubious of that diagnosis, he declined recommended treatment. Nonetheless, he is interested in knowing exactly what is happening to his nails.

He denies personal or family history of skin cancer and of excessive sun exposure. He reports that several maternal family members have similar nail changes.

Was Declining Treatment a Bad Idea?

EXAMINATION
Seven of the patient’s 10 fingernails demonstrate linear brown streaks that uniformly average 1.5 to 2 mm in width. The streaks run the length of the nail, with no involvement of the adjacent cuticle. Some are darker than others.

The patient has type V skin with no evidence of excessive sun damage.

What’s the diagnosis?

 

 

DISCUSSION
Fortunately, this patient’s problem is benign and likely to remain so. Termed longitudinal (or linear) melanonychia (LM), these changes are seen in nearly all African-Americans older than 50 (although it is not uncommon for the condition to develop in the third decade of life). Other populations with dark skin are also at risk for LM, albeit at far lower rates. In white populations, the incidence is around 0.5% to 1%.

LM is caused by activation and proliferation of melanocytes in the nail matrix; they are focally incorporated into the nail plate as onychocytes that grow out with the nail. Typically 1 to 3 mm in uniform width, the streaks of LM range from tan to dark brown and can be solitary or multiple in a given nail.

As mentioned, LM is entirely benign, with almost no potential for malignant transformation. However, two notes of caution are in order: First, although African-American persons generally have very low risk for melanoma, the malignancy tends to manifest in this population in areas with the least pigment (eg, palms, soles, oral cavities, nail beds—unusual locations for most other racial groups). Second, the prognosis for these types of melanomas is poor; most patients and providers are unaware of them until an advanced stage that typically includes metastasis.

Therefore, in patients with skin of color, new or changing lesions in the nail bed must be evaluated by a knowledgeable dermatology provider, who may choose to biopsy the proximal aspect of the lesion to rule out cancer. Of course, any such lesion in a white person needs to be monitored carefully as well, since linear melanonychia is relatively uncommon in this group. Changes in the width, color, or border should cause concern, as should extension of the darker color onto the adjacent cuticle.

The differential for linear discoloration in nails or nail beds includes foreign body, warts, benign tumors (eg, nevi), glomus tumors (which are usually painful), and of course, fungal, mold, or yeast infections.

TAKE-HOME LEARNING POINTS

  • Longitudinal (or linear) melanonychia (LM) is quite common in African-Americans, approaching a prevalence of 100% in those older than 50.
  • Having multiple LMs in more than one finger is common in this population.
  • However, a new or changing subungual lesion bears close monitoring, or even biopsy, by an experienced dermatology provider.
  • Although African-Americans rarely develop melanoma, when they do, it’s often in the least pigmented areas (eg, palms, soles, mouth, and nails).
  • The prognosis for proven melanoma in African-American patients is poor, making close monitoring a necessity.

 

Publications
Topics
Sections

A 50-year-old African-American man is referred to dermatology by his primary care provider for evaluation of colored stripes in most of his fingernails. These have been present, without change, for most of his adult life.

The patient has been told these changes probably represent fungal infection, but being dubious of that diagnosis, he declined recommended treatment. Nonetheless, he is interested in knowing exactly what is happening to his nails.

He denies personal or family history of skin cancer and of excessive sun exposure. He reports that several maternal family members have similar nail changes.

Was Declining Treatment a Bad Idea?

EXAMINATION
Seven of the patient’s 10 fingernails demonstrate linear brown streaks that uniformly average 1.5 to 2 mm in width. The streaks run the length of the nail, with no involvement of the adjacent cuticle. Some are darker than others.

The patient has type V skin with no evidence of excessive sun damage.

What’s the diagnosis?

 

 

DISCUSSION
Fortunately, this patient’s problem is benign and likely to remain so. Termed longitudinal (or linear) melanonychia (LM), these changes are seen in nearly all African-Americans older than 50 (although it is not uncommon for the condition to develop in the third decade of life). Other populations with dark skin are also at risk for LM, albeit at far lower rates. In white populations, the incidence is around 0.5% to 1%.

LM is caused by activation and proliferation of melanocytes in the nail matrix; they are focally incorporated into the nail plate as onychocytes that grow out with the nail. Typically 1 to 3 mm in uniform width, the streaks of LM range from tan to dark brown and can be solitary or multiple in a given nail.

As mentioned, LM is entirely benign, with almost no potential for malignant transformation. However, two notes of caution are in order: First, although African-American persons generally have very low risk for melanoma, the malignancy tends to manifest in this population in areas with the least pigment (eg, palms, soles, oral cavities, nail beds—unusual locations for most other racial groups). Second, the prognosis for these types of melanomas is poor; most patients and providers are unaware of them until an advanced stage that typically includes metastasis.

Therefore, in patients with skin of color, new or changing lesions in the nail bed must be evaluated by a knowledgeable dermatology provider, who may choose to biopsy the proximal aspect of the lesion to rule out cancer. Of course, any such lesion in a white person needs to be monitored carefully as well, since linear melanonychia is relatively uncommon in this group. Changes in the width, color, or border should cause concern, as should extension of the darker color onto the adjacent cuticle.

The differential for linear discoloration in nails or nail beds includes foreign body, warts, benign tumors (eg, nevi), glomus tumors (which are usually painful), and of course, fungal, mold, or yeast infections.

TAKE-HOME LEARNING POINTS

  • Longitudinal (or linear) melanonychia (LM) is quite common in African-Americans, approaching a prevalence of 100% in those older than 50.
  • Having multiple LMs in more than one finger is common in this population.
  • However, a new or changing subungual lesion bears close monitoring, or even biopsy, by an experienced dermatology provider.
  • Although African-Americans rarely develop melanoma, when they do, it’s often in the least pigmented areas (eg, palms, soles, mouth, and nails).
  • The prognosis for proven melanoma in African-American patients is poor, making close monitoring a necessity.

 

A 50-year-old African-American man is referred to dermatology by his primary care provider for evaluation of colored stripes in most of his fingernails. These have been present, without change, for most of his adult life.

The patient has been told these changes probably represent fungal infection, but being dubious of that diagnosis, he declined recommended treatment. Nonetheless, he is interested in knowing exactly what is happening to his nails.

He denies personal or family history of skin cancer and of excessive sun exposure. He reports that several maternal family members have similar nail changes.

Was Declining Treatment a Bad Idea?

EXAMINATION
Seven of the patient’s 10 fingernails demonstrate linear brown streaks that uniformly average 1.5 to 2 mm in width. The streaks run the length of the nail, with no involvement of the adjacent cuticle. Some are darker than others.

The patient has type V skin with no evidence of excessive sun damage.

What’s the diagnosis?

 

 

DISCUSSION
Fortunately, this patient’s problem is benign and likely to remain so. Termed longitudinal (or linear) melanonychia (LM), these changes are seen in nearly all African-Americans older than 50 (although it is not uncommon for the condition to develop in the third decade of life). Other populations with dark skin are also at risk for LM, albeit at far lower rates. In white populations, the incidence is around 0.5% to 1%.

LM is caused by activation and proliferation of melanocytes in the nail matrix; they are focally incorporated into the nail plate as onychocytes that grow out with the nail. Typically 1 to 3 mm in uniform width, the streaks of LM range from tan to dark brown and can be solitary or multiple in a given nail.

As mentioned, LM is entirely benign, with almost no potential for malignant transformation. However, two notes of caution are in order: First, although African-American persons generally have very low risk for melanoma, the malignancy tends to manifest in this population in areas with the least pigment (eg, palms, soles, oral cavities, nail beds—unusual locations for most other racial groups). Second, the prognosis for these types of melanomas is poor; most patients and providers are unaware of them until an advanced stage that typically includes metastasis.

Therefore, in patients with skin of color, new or changing lesions in the nail bed must be evaluated by a knowledgeable dermatology provider, who may choose to biopsy the proximal aspect of the lesion to rule out cancer. Of course, any such lesion in a white person needs to be monitored carefully as well, since linear melanonychia is relatively uncommon in this group. Changes in the width, color, or border should cause concern, as should extension of the darker color onto the adjacent cuticle.

The differential for linear discoloration in nails or nail beds includes foreign body, warts, benign tumors (eg, nevi), glomus tumors (which are usually painful), and of course, fungal, mold, or yeast infections.

TAKE-HOME LEARNING POINTS

  • Longitudinal (or linear) melanonychia (LM) is quite common in African-Americans, approaching a prevalence of 100% in those older than 50.
  • Having multiple LMs in more than one finger is common in this population.
  • However, a new or changing subungual lesion bears close monitoring, or even biopsy, by an experienced dermatology provider.
  • Although African-Americans rarely develop melanoma, when they do, it’s often in the least pigmented areas (eg, palms, soles, mouth, and nails).
  • The prognosis for proven melanoma in African-American patients is poor, making close monitoring a necessity.

 

Publications
Publications
Topics
Article Type
Display Headline
Was Declining Treatment a Bad Idea?
Display Headline
Was Declining Treatment a Bad Idea?
Sections
Disallow All Ads
Content Gating
No Gating (article Unlocked/Free)
Alternative CME
Disqus Comments
Default
Gate On Date
Tue, 04/09/2019 - 09:30
Un-Gate On Date
Tue, 04/09/2019 - 09:30
Use ProPublica
CFC Schedule Remove Status
Tue, 04/09/2019 - 09:30
Hide sidebar & use full width
render the right sidebar.

Recognizing the Scale of the Problem

Article Type
Changed
Fri, 03/29/2019 - 08:19
Display Headline
Recognizing the Scale of the Problem

For more than 2 years, this 36-year-old woman has had a slightly itchy rash that waxes and wanes on her posterior neck. She has consulted several primary care providers and received multiple diagnoses, the most consistent of which has been fungal infection. However, despite use of a variety of antifungal creams (nystatin, clotrimazole, and combination clotrimazole/betamethasone), a 1-month course of oral terbinafine, and OTC tolnaftate, no improvement has occurred.

The patient asserts that she is otherwise in good health, with no joint pain or fever and no history of recent health crises. Family history is free of dermatologic complaints except for psoriasis in her father.

Recognizing the Scale of the Problem

EXAMINATION
A pink plaque with white, fairly adherent scale covers most of the patient’s posterior neck/upper midline back. When a 3-mm section of scaling is peeled away, 2 tiny dots of pinpoint bleeding are immediately noted.

The rest of her scalp is free of any such changes, as are her elbows and knees. But a similar rash is seen in the upper intergluteal area, and 3 of 10 fingernails are mildly pitted.

What’s the diagnosis?

 

 

DISCUSSION
Psoriasis vulgaris (common psoriasis) affects around 3% of the white population in this country. That incidence almost doubles in northern Europe and Scandinavia.

Psoriasis is so common that you should expect to see it regularly; the important question is not “Will you see it?” but rather “Will you know it when you see it?” Sometimes the various clinical elements of psoriasis must be sought, and those dots connected, as this case demonstrates effectively.

For one thing, the nape of the neck is commonly affected, especially in women. It is pure speculation, but one imagines that the heat and sweat associated with longer hair might contribute to this predilection.

The pink color, whitish scale, and pinpoint bleeding (termed the Auspitz sign) all corroborate the diagnosis, as does the positive family history and nail pitting. The intergluteal involvement was the icing on the cake; this is seen in only 2 common conditions: psoriasis and seborrhea.

The lesson? Even though psoriasis is supposed to appear on elbows, knees, and other extensor surfaces, sometimes it breaks the rules. The posterior neck was the primary area of involvement in this case, but sometimes psoriasis is completely confined to the scalp or the palms. And, of course, there are different types of psoriasis, some of which bear scant resemblance to psoriasis vulgaris. That’s where biopsies and/or referrals prove to be useful.

It is true that this patient’s rash could have had a fungal origin. When in doubt, however, a punch or shave biopsy would most likely settle the matter, since the histologic picture is usually pathognomic.

TAKE-HOME LEARNING POINTS

  • Psoriasis is often be easy to diagnose—but just as often, it takes a bit of detective work.
  • This “investigation” consists of looking for and asking about findings that could corroborate the diagnosis.
  • The morphology of the neck lesion, as well as the Auspitz sign, nail pitting, intergluteal involvement, and family history in this case all served quite well to establish the diagnosis of psoriasis.
  • It is helpful to remember how utterly common psoriasis is, affecting around 10,000,000 Americans.
Publications
Topics
Sections

For more than 2 years, this 36-year-old woman has had a slightly itchy rash that waxes and wanes on her posterior neck. She has consulted several primary care providers and received multiple diagnoses, the most consistent of which has been fungal infection. However, despite use of a variety of antifungal creams (nystatin, clotrimazole, and combination clotrimazole/betamethasone), a 1-month course of oral terbinafine, and OTC tolnaftate, no improvement has occurred.

The patient asserts that she is otherwise in good health, with no joint pain or fever and no history of recent health crises. Family history is free of dermatologic complaints except for psoriasis in her father.

Recognizing the Scale of the Problem

EXAMINATION
A pink plaque with white, fairly adherent scale covers most of the patient’s posterior neck/upper midline back. When a 3-mm section of scaling is peeled away, 2 tiny dots of pinpoint bleeding are immediately noted.

The rest of her scalp is free of any such changes, as are her elbows and knees. But a similar rash is seen in the upper intergluteal area, and 3 of 10 fingernails are mildly pitted.

What’s the diagnosis?

 

 

DISCUSSION
Psoriasis vulgaris (common psoriasis) affects around 3% of the white population in this country. That incidence almost doubles in northern Europe and Scandinavia.

Psoriasis is so common that you should expect to see it regularly; the important question is not “Will you see it?” but rather “Will you know it when you see it?” Sometimes the various clinical elements of psoriasis must be sought, and those dots connected, as this case demonstrates effectively.

For one thing, the nape of the neck is commonly affected, especially in women. It is pure speculation, but one imagines that the heat and sweat associated with longer hair might contribute to this predilection.

The pink color, whitish scale, and pinpoint bleeding (termed the Auspitz sign) all corroborate the diagnosis, as does the positive family history and nail pitting. The intergluteal involvement was the icing on the cake; this is seen in only 2 common conditions: psoriasis and seborrhea.

The lesson? Even though psoriasis is supposed to appear on elbows, knees, and other extensor surfaces, sometimes it breaks the rules. The posterior neck was the primary area of involvement in this case, but sometimes psoriasis is completely confined to the scalp or the palms. And, of course, there are different types of psoriasis, some of which bear scant resemblance to psoriasis vulgaris. That’s where biopsies and/or referrals prove to be useful.

It is true that this patient’s rash could have had a fungal origin. When in doubt, however, a punch or shave biopsy would most likely settle the matter, since the histologic picture is usually pathognomic.

TAKE-HOME LEARNING POINTS

  • Psoriasis is often be easy to diagnose—but just as often, it takes a bit of detective work.
  • This “investigation” consists of looking for and asking about findings that could corroborate the diagnosis.
  • The morphology of the neck lesion, as well as the Auspitz sign, nail pitting, intergluteal involvement, and family history in this case all served quite well to establish the diagnosis of psoriasis.
  • It is helpful to remember how utterly common psoriasis is, affecting around 10,000,000 Americans.

For more than 2 years, this 36-year-old woman has had a slightly itchy rash that waxes and wanes on her posterior neck. She has consulted several primary care providers and received multiple diagnoses, the most consistent of which has been fungal infection. However, despite use of a variety of antifungal creams (nystatin, clotrimazole, and combination clotrimazole/betamethasone), a 1-month course of oral terbinafine, and OTC tolnaftate, no improvement has occurred.

The patient asserts that she is otherwise in good health, with no joint pain or fever and no history of recent health crises. Family history is free of dermatologic complaints except for psoriasis in her father.

Recognizing the Scale of the Problem

EXAMINATION
A pink plaque with white, fairly adherent scale covers most of the patient’s posterior neck/upper midline back. When a 3-mm section of scaling is peeled away, 2 tiny dots of pinpoint bleeding are immediately noted.

The rest of her scalp is free of any such changes, as are her elbows and knees. But a similar rash is seen in the upper intergluteal area, and 3 of 10 fingernails are mildly pitted.

What’s the diagnosis?

 

 

DISCUSSION
Psoriasis vulgaris (common psoriasis) affects around 3% of the white population in this country. That incidence almost doubles in northern Europe and Scandinavia.

Psoriasis is so common that you should expect to see it regularly; the important question is not “Will you see it?” but rather “Will you know it when you see it?” Sometimes the various clinical elements of psoriasis must be sought, and those dots connected, as this case demonstrates effectively.

For one thing, the nape of the neck is commonly affected, especially in women. It is pure speculation, but one imagines that the heat and sweat associated with longer hair might contribute to this predilection.

The pink color, whitish scale, and pinpoint bleeding (termed the Auspitz sign) all corroborate the diagnosis, as does the positive family history and nail pitting. The intergluteal involvement was the icing on the cake; this is seen in only 2 common conditions: psoriasis and seborrhea.

The lesson? Even though psoriasis is supposed to appear on elbows, knees, and other extensor surfaces, sometimes it breaks the rules. The posterior neck was the primary area of involvement in this case, but sometimes psoriasis is completely confined to the scalp or the palms. And, of course, there are different types of psoriasis, some of which bear scant resemblance to psoriasis vulgaris. That’s where biopsies and/or referrals prove to be useful.

It is true that this patient’s rash could have had a fungal origin. When in doubt, however, a punch or shave biopsy would most likely settle the matter, since the histologic picture is usually pathognomic.

TAKE-HOME LEARNING POINTS

  • Psoriasis is often be easy to diagnose—but just as often, it takes a bit of detective work.
  • This “investigation” consists of looking for and asking about findings that could corroborate the diagnosis.
  • The morphology of the neck lesion, as well as the Auspitz sign, nail pitting, intergluteal involvement, and family history in this case all served quite well to establish the diagnosis of psoriasis.
  • It is helpful to remember how utterly common psoriasis is, affecting around 10,000,000 Americans.
Publications
Publications
Topics
Article Type
Display Headline
Recognizing the Scale of the Problem
Display Headline
Recognizing the Scale of the Problem
Sections
Disallow All Ads
Content Gating
No Gating (article Unlocked/Free)
Alternative CME
Disqus Comments
Default
Gate On Date
Fri, 03/29/2019 - 08:00
Un-Gate On Date
Fri, 03/29/2019 - 08:00
Use ProPublica
CFC Schedule Remove Status
Fri, 03/29/2019 - 08:00
Hide sidebar & use full width
render the right sidebar.

There’s No Doubt: Blame the Drug

Article Type
Changed
Thu, 03/21/2019 - 10:55
Display Headline
There’s No Doubt: Blame the Drug

It’s taken 2 years for family members to convince this 50-year-old black woman to consult dermatology about dark circles on her arms and trunk. The asymptomatic lesions were only faintly discolored at manifestation but have darkened with each recurrence. The 6 lesions she currently has are in the exact locations they originally manifested in.

Convinced she had “ringworm,” the patient tried several different antifungal creams, to no avail. Then her primary care provider prescribed a 2-month course of oral antifungal medication (terbinafine)—again, with no change to the lesions.

The patient is otherwise healthy except for osteoarthritis, for which she takes ibuprofen (800 mg 2 or 3 times a day). She denies any history of seizure, chronic urinary tract infection, or other chronic infection.

There’s No Doubt: Blame the Drug

EXAMINATION
The 6 perfectly round, uniformly pigmented, dark brown macular lesions on the patient’s arms and trunk range from 2 to 5 cm in diameter. A faint, rusty brown halo can be seen around the periphery of the lesions, giving them a targetoid look. The surfaces of the lesions are completely smooth.

What’s the diagnosis?

 

 

DISCUSSION
The list of dark brown, round macules that come and go in the exact same locations is a short one—in fact, it only includes fixed drug eruption (FDE). This is a unique adverse reaction to one of several medications; common culprits are ibuprofen or other NSAIDs, aspirin, members of the sulfa family, penicillin, and several antiseizure medications.

The exact mechanism for this reaction is unknown, but it does appear to involve the localized production of cytokines. There is a range of morphologic presentations for FDE: some lesions are more targetoid than others; some are darker (especially in patients with darker skin) while others are more pink.

Furthermore, many patients develop additional lesions over time. A few, on the other hand, will reach a point at which they stop reacting to the offending product.

Of all the drugs causing FDE, sulfa-based products are among the more consistent offenders. Thus, we always ask about chronic urinary tract infections.

TAKE-HOME LEARNING POINTS

  • Fixed drug eruptions (FDE) usually manifest with round, targetoid macules that are occasionally blistery.
  • The lesions recur or darken with each “challenge” from the drug, which can be one of several potential offenders.
  • The fact that the lesions keep recurring in the same location (ie, fixed) is pathognomic for FDE.
  • FDE lesions can occur almost anywhere on the body—even on palms or soles.
Publications
Topics
Sections

It’s taken 2 years for family members to convince this 50-year-old black woman to consult dermatology about dark circles on her arms and trunk. The asymptomatic lesions were only faintly discolored at manifestation but have darkened with each recurrence. The 6 lesions she currently has are in the exact locations they originally manifested in.

Convinced she had “ringworm,” the patient tried several different antifungal creams, to no avail. Then her primary care provider prescribed a 2-month course of oral antifungal medication (terbinafine)—again, with no change to the lesions.

The patient is otherwise healthy except for osteoarthritis, for which she takes ibuprofen (800 mg 2 or 3 times a day). She denies any history of seizure, chronic urinary tract infection, or other chronic infection.

There’s No Doubt: Blame the Drug

EXAMINATION
The 6 perfectly round, uniformly pigmented, dark brown macular lesions on the patient’s arms and trunk range from 2 to 5 cm in diameter. A faint, rusty brown halo can be seen around the periphery of the lesions, giving them a targetoid look. The surfaces of the lesions are completely smooth.

What’s the diagnosis?

 

 

DISCUSSION
The list of dark brown, round macules that come and go in the exact same locations is a short one—in fact, it only includes fixed drug eruption (FDE). This is a unique adverse reaction to one of several medications; common culprits are ibuprofen or other NSAIDs, aspirin, members of the sulfa family, penicillin, and several antiseizure medications.

The exact mechanism for this reaction is unknown, but it does appear to involve the localized production of cytokines. There is a range of morphologic presentations for FDE: some lesions are more targetoid than others; some are darker (especially in patients with darker skin) while others are more pink.

Furthermore, many patients develop additional lesions over time. A few, on the other hand, will reach a point at which they stop reacting to the offending product.

Of all the drugs causing FDE, sulfa-based products are among the more consistent offenders. Thus, we always ask about chronic urinary tract infections.

TAKE-HOME LEARNING POINTS

  • Fixed drug eruptions (FDE) usually manifest with round, targetoid macules that are occasionally blistery.
  • The lesions recur or darken with each “challenge” from the drug, which can be one of several potential offenders.
  • The fact that the lesions keep recurring in the same location (ie, fixed) is pathognomic for FDE.
  • FDE lesions can occur almost anywhere on the body—even on palms or soles.

It’s taken 2 years for family members to convince this 50-year-old black woman to consult dermatology about dark circles on her arms and trunk. The asymptomatic lesions were only faintly discolored at manifestation but have darkened with each recurrence. The 6 lesions she currently has are in the exact locations they originally manifested in.

Convinced she had “ringworm,” the patient tried several different antifungal creams, to no avail. Then her primary care provider prescribed a 2-month course of oral antifungal medication (terbinafine)—again, with no change to the lesions.

The patient is otherwise healthy except for osteoarthritis, for which she takes ibuprofen (800 mg 2 or 3 times a day). She denies any history of seizure, chronic urinary tract infection, or other chronic infection.

There’s No Doubt: Blame the Drug

EXAMINATION
The 6 perfectly round, uniformly pigmented, dark brown macular lesions on the patient’s arms and trunk range from 2 to 5 cm in diameter. A faint, rusty brown halo can be seen around the periphery of the lesions, giving them a targetoid look. The surfaces of the lesions are completely smooth.

What’s the diagnosis?

 

 

DISCUSSION
The list of dark brown, round macules that come and go in the exact same locations is a short one—in fact, it only includes fixed drug eruption (FDE). This is a unique adverse reaction to one of several medications; common culprits are ibuprofen or other NSAIDs, aspirin, members of the sulfa family, penicillin, and several antiseizure medications.

The exact mechanism for this reaction is unknown, but it does appear to involve the localized production of cytokines. There is a range of morphologic presentations for FDE: some lesions are more targetoid than others; some are darker (especially in patients with darker skin) while others are more pink.

Furthermore, many patients develop additional lesions over time. A few, on the other hand, will reach a point at which they stop reacting to the offending product.

Of all the drugs causing FDE, sulfa-based products are among the more consistent offenders. Thus, we always ask about chronic urinary tract infections.

TAKE-HOME LEARNING POINTS

  • Fixed drug eruptions (FDE) usually manifest with round, targetoid macules that are occasionally blistery.
  • The lesions recur or darken with each “challenge” from the drug, which can be one of several potential offenders.
  • The fact that the lesions keep recurring in the same location (ie, fixed) is pathognomic for FDE.
  • FDE lesions can occur almost anywhere on the body—even on palms or soles.
Publications
Publications
Topics
Article Type
Display Headline
There’s No Doubt: Blame the Drug
Display Headline
There’s No Doubt: Blame the Drug
Sections
Disallow All Ads
Content Gating
No Gating (article Unlocked/Free)
Alternative CME
Disqus Comments
Default
Gate On Date
Thu, 03/14/2019 - 09:30
Un-Gate On Date
Thu, 03/14/2019 - 09:30
Use ProPublica
CFC Schedule Remove Status
Thu, 03/14/2019 - 09:30
Hide sidebar & use full width
render the right sidebar.

The Case of the Disappearing Eyebrows

Article Type
Changed
Thu, 02/28/2019 - 09:54
Display Headline
The Case of the Disappearing Eyebrows

When this 45-year-old man was in third grade, he began to notice areas of hair loss in his scalp. The affected area was always round and the hair loss complete—but it would grow back entirely within weeks to months. There was never any rash or discomfort associated with these changes.

Since then, he has experienced numerous similar episodes of focal hair loss, sometimes in the beard, sometimes on the arms or legs, and most recently, in his eyebrows. Again, no symptoms accompany the process.

Although his personal health history is relatively uneventful, his family has not been as fortunate. There are numerous cases of lupus, rheumatoid arthritis, diabetes, and thyroid disease.

The Case of the Disappearing Eyebrows

EXAMINATION
There are sharply defined, crescent-shaped, 2.5-cm divots at the superior borders of both eyebrows in which every hair is gone. No redness, swelling, or scaling are seen or felt, and there is no detectable adenopathy in the region.

Examination of hair-bearing regions reveals no other areas of hair loss.

What’s the diagnosis?

 

 

DISCUSSION
This case illustrates several variants of an extremely common condition: alopecia areata (AA), literally translated as “hair loss confined to a particular localized area or areas.” Although AA is more common in adults, it often affects children. And though the scalp is by far the most commonly affected area, AA can cause hair loss anywhere on the body.

In general, the earlier the onset, the more likely the problem is to become recurrent or even progressive. In rare instances, a patient can develop alopecia totalis, in which there is permanent and total scalp hair loss, or even alopecia universalis, the permanent loss of every hair on the entire body. Other predictors of a poor prognosis include extensive involvement of the scalp, especially the periphery (termed ophiasis), and a history of atopy.

Much research has been done on the underlying pathology of AA, as well as potential remedies. This has proven, beyond any doubt, that the problem is autoimmune and tends to run in families (as with other autoimmune diseases), suggesting a hereditary basis. We also know that medications that dampen this autoimmune process, such as steroids and biologics, are useful but not always safe or practical.

What we don’t really know is what triggers an actual attack. My observation, based on 35 years of dermatology practice, is that stress often plays a part—but I’d be hard pressed to prove that, and it wouldn’t be very useful even if I could. This patient adamantly agreed that stress was the trigger for his AA.

The differential for AA includes tinea capitis, discoid lupus, and lichen planopilaris. This patient’s multiple episodes over decades made the correct diagnosis clear. However, when the diagnosis is in doubt—other items in the differential commonly affect the scalp, although they may also manifest with redness, scaling, or swelling—a punch biopsy may be necessary to sort through the possibilities. Care must be taken to enter the skin parallel to hair follicles when the sample is removed.

Many treatments have been tried for AA, but none are reliably effective. In the vast majority of patients, the problem resolves itself. Treatments to try when needed include topical steroids, intralesional steroids, and topical immune stimulators (eg, squaric acid or dinitrochlorobenzene)—all of which have their limitations.

TAKE-HOME LEARNING POINTS

  • Alopecia areata (AA) is more common in adults but can affect children as well.
  • In rare cases, the patient can experience permanent total hair loss on the scalp (alopecia totalis) or the entire body (alopecia universalis).
  • Because AA is an autoimmune disease, steroids and biologics may be useful treatments—but they are not always safe or practical.
  • In a majority of patients with AA, the problem will resolve on its own.
Publications
Topics
Sections

When this 45-year-old man was in third grade, he began to notice areas of hair loss in his scalp. The affected area was always round and the hair loss complete—but it would grow back entirely within weeks to months. There was never any rash or discomfort associated with these changes.

Since then, he has experienced numerous similar episodes of focal hair loss, sometimes in the beard, sometimes on the arms or legs, and most recently, in his eyebrows. Again, no symptoms accompany the process.

Although his personal health history is relatively uneventful, his family has not been as fortunate. There are numerous cases of lupus, rheumatoid arthritis, diabetes, and thyroid disease.

The Case of the Disappearing Eyebrows

EXAMINATION
There are sharply defined, crescent-shaped, 2.5-cm divots at the superior borders of both eyebrows in which every hair is gone. No redness, swelling, or scaling are seen or felt, and there is no detectable adenopathy in the region.

Examination of hair-bearing regions reveals no other areas of hair loss.

What’s the diagnosis?

 

 

DISCUSSION
This case illustrates several variants of an extremely common condition: alopecia areata (AA), literally translated as “hair loss confined to a particular localized area or areas.” Although AA is more common in adults, it often affects children. And though the scalp is by far the most commonly affected area, AA can cause hair loss anywhere on the body.

In general, the earlier the onset, the more likely the problem is to become recurrent or even progressive. In rare instances, a patient can develop alopecia totalis, in which there is permanent and total scalp hair loss, or even alopecia universalis, the permanent loss of every hair on the entire body. Other predictors of a poor prognosis include extensive involvement of the scalp, especially the periphery (termed ophiasis), and a history of atopy.

Much research has been done on the underlying pathology of AA, as well as potential remedies. This has proven, beyond any doubt, that the problem is autoimmune and tends to run in families (as with other autoimmune diseases), suggesting a hereditary basis. We also know that medications that dampen this autoimmune process, such as steroids and biologics, are useful but not always safe or practical.

What we don’t really know is what triggers an actual attack. My observation, based on 35 years of dermatology practice, is that stress often plays a part—but I’d be hard pressed to prove that, and it wouldn’t be very useful even if I could. This patient adamantly agreed that stress was the trigger for his AA.

The differential for AA includes tinea capitis, discoid lupus, and lichen planopilaris. This patient’s multiple episodes over decades made the correct diagnosis clear. However, when the diagnosis is in doubt—other items in the differential commonly affect the scalp, although they may also manifest with redness, scaling, or swelling—a punch biopsy may be necessary to sort through the possibilities. Care must be taken to enter the skin parallel to hair follicles when the sample is removed.

Many treatments have been tried for AA, but none are reliably effective. In the vast majority of patients, the problem resolves itself. Treatments to try when needed include topical steroids, intralesional steroids, and topical immune stimulators (eg, squaric acid or dinitrochlorobenzene)—all of which have their limitations.

TAKE-HOME LEARNING POINTS

  • Alopecia areata (AA) is more common in adults but can affect children as well.
  • In rare cases, the patient can experience permanent total hair loss on the scalp (alopecia totalis) or the entire body (alopecia universalis).
  • Because AA is an autoimmune disease, steroids and biologics may be useful treatments—but they are not always safe or practical.
  • In a majority of patients with AA, the problem will resolve on its own.

When this 45-year-old man was in third grade, he began to notice areas of hair loss in his scalp. The affected area was always round and the hair loss complete—but it would grow back entirely within weeks to months. There was never any rash or discomfort associated with these changes.

Since then, he has experienced numerous similar episodes of focal hair loss, sometimes in the beard, sometimes on the arms or legs, and most recently, in his eyebrows. Again, no symptoms accompany the process.

Although his personal health history is relatively uneventful, his family has not been as fortunate. There are numerous cases of lupus, rheumatoid arthritis, diabetes, and thyroid disease.

The Case of the Disappearing Eyebrows

EXAMINATION
There are sharply defined, crescent-shaped, 2.5-cm divots at the superior borders of both eyebrows in which every hair is gone. No redness, swelling, or scaling are seen or felt, and there is no detectable adenopathy in the region.

Examination of hair-bearing regions reveals no other areas of hair loss.

What’s the diagnosis?

 

 

DISCUSSION
This case illustrates several variants of an extremely common condition: alopecia areata (AA), literally translated as “hair loss confined to a particular localized area or areas.” Although AA is more common in adults, it often affects children. And though the scalp is by far the most commonly affected area, AA can cause hair loss anywhere on the body.

In general, the earlier the onset, the more likely the problem is to become recurrent or even progressive. In rare instances, a patient can develop alopecia totalis, in which there is permanent and total scalp hair loss, or even alopecia universalis, the permanent loss of every hair on the entire body. Other predictors of a poor prognosis include extensive involvement of the scalp, especially the periphery (termed ophiasis), and a history of atopy.

Much research has been done on the underlying pathology of AA, as well as potential remedies. This has proven, beyond any doubt, that the problem is autoimmune and tends to run in families (as with other autoimmune diseases), suggesting a hereditary basis. We also know that medications that dampen this autoimmune process, such as steroids and biologics, are useful but not always safe or practical.

What we don’t really know is what triggers an actual attack. My observation, based on 35 years of dermatology practice, is that stress often plays a part—but I’d be hard pressed to prove that, and it wouldn’t be very useful even if I could. This patient adamantly agreed that stress was the trigger for his AA.

The differential for AA includes tinea capitis, discoid lupus, and lichen planopilaris. This patient’s multiple episodes over decades made the correct diagnosis clear. However, when the diagnosis is in doubt—other items in the differential commonly affect the scalp, although they may also manifest with redness, scaling, or swelling—a punch biopsy may be necessary to sort through the possibilities. Care must be taken to enter the skin parallel to hair follicles when the sample is removed.

Many treatments have been tried for AA, but none are reliably effective. In the vast majority of patients, the problem resolves itself. Treatments to try when needed include topical steroids, intralesional steroids, and topical immune stimulators (eg, squaric acid or dinitrochlorobenzene)—all of which have their limitations.

TAKE-HOME LEARNING POINTS

  • Alopecia areata (AA) is more common in adults but can affect children as well.
  • In rare cases, the patient can experience permanent total hair loss on the scalp (alopecia totalis) or the entire body (alopecia universalis).
  • Because AA is an autoimmune disease, steroids and biologics may be useful treatments—but they are not always safe or practical.
  • In a majority of patients with AA, the problem will resolve on its own.
Publications
Publications
Topics
Article Type
Display Headline
The Case of the Disappearing Eyebrows
Display Headline
The Case of the Disappearing Eyebrows
Sections
Disallow All Ads
Content Gating
No Gating (article Unlocked/Free)
Alternative CME
Disqus Comments
Default
Gate On Date
Thu, 02/28/2019 - 09:30
Un-Gate On Date
Thu, 02/28/2019 - 09:30
Use ProPublica
CFC Schedule Remove Status
Thu, 02/28/2019 - 09:30
Hide sidebar & use full width
render the right sidebar.

A Faux Fungal Affliction

Article Type
Changed
Wed, 11/20/2019 - 12:01
Display Headline
A Faux Fungal Affliction

A 45-year-old woman is referred to dermatology for a “fungal infection” that has failed to respond to the following treatments: topical clotrimazole cream, topical miconazole cream, a 30-day course of oral terbinafine (250 mg/d), and a 2-month course of oral griseofulvin (unknown dose). The lesions are completely asymptomatic but quite worrisome to the patient since they manifested 6 months ago.

She has consulted at least 6 different providers—none of whom was a dermatologist but all of whom were certain of the diagnosis and thus felt no need to refer the patient. However, the passage of time and trail of ineffective treatments finally prompts the (albeit reluctant) decision to send the patient to dermatology.

On questioning, she denies any serious health problems, such as diabetes or immunosuppression. She has had no contact with any animals or children.

A Faux Fungal Affliction

EXAMINATION
The lesions in question total 6; all are uniformly purplish brown, round, and macular, and they range from 5 mm to more than 3 cm. Most are located on the bilateral popliteal areas. The lesions have sharp, well-defined margins. Several have faintly raised papular margins that give the centers a slightly concave appearance.

Palpation reveals the complete absence of any surface disturbance, such as scaling or erosion. Thus, no KOH prep can be performed to check for fungal elements. Instead, a shave biopsy is performed, the results of which show a sawtooth-patterned lymphocytic infiltrate obliterating the normally smooth undulating dermoepidermal junction.

What’s the diagnosis?

 

 

DISCUSSION
This case effectively demonstrates the principle that, when confronted with round or annular lesions, some providers will rely on the diagnosis of “fungal” even when evidence (eg, failed treatment attempts) suggests otherwise. What that nonresponse should do is signal the need for an expanded differential—that is, a consideration of other diagnostic possibilities. This is a bedrock principle in every medical specialty, not just in dermatology.

In this case, the biopsy results clearly pointed to the correct diagnosis of lichen planus (LP), a common dermatosis well known to present in annular morphology. LP is a benign process, albeit one that is occasionally quite bothersome (eg, itching) and, rarely, widespread. LP’s more typical distribution is on volar wrists, in the sacral areas, and occasionally on genitals, so the inability to make a visual diagnosis in this case is forgivable.

Although LP’s etiology is unfortunately unknown, what is known is how to treat it: with topical steroids when necessary or “tincture of time,” as in this patient’s asymptomatic case. LP typically resolves on its own, and it has no worrisome import or connections to more serious disease.

But as always, the first step to correct diagnosis is to consider letting go of the old diagnosis—fungal infection—which was clearly incorrect given the lack of response to numerous antifungals. The second step is to consider other possibilities, which would include lichen planus, psoriasis, granuloma annulare, tinea versicolor, and necrobiosis. The third step is to perform a biopsy, which would establish the correct diagnosis with certainty and in turn, dictate correct treatment.

TAKE-HOME LEARNING POINTS

  • There is an extensive differential for round or annular skin lesions that includes many nonfungal causes.
  • When antifungals fail to help, consider other diagnostic possibilities.
  • Perform a biopsy when a visual diagnosis is not possible.
  • Lichen planus (LP) is a common benign inflammatory skin condition that can present with annular lesions.
Publications
Topics
Sections

A 45-year-old woman is referred to dermatology for a “fungal infection” that has failed to respond to the following treatments: topical clotrimazole cream, topical miconazole cream, a 30-day course of oral terbinafine (250 mg/d), and a 2-month course of oral griseofulvin (unknown dose). The lesions are completely asymptomatic but quite worrisome to the patient since they manifested 6 months ago.

She has consulted at least 6 different providers—none of whom was a dermatologist but all of whom were certain of the diagnosis and thus felt no need to refer the patient. However, the passage of time and trail of ineffective treatments finally prompts the (albeit reluctant) decision to send the patient to dermatology.

On questioning, she denies any serious health problems, such as diabetes or immunosuppression. She has had no contact with any animals or children.

A Faux Fungal Affliction

EXAMINATION
The lesions in question total 6; all are uniformly purplish brown, round, and macular, and they range from 5 mm to more than 3 cm. Most are located on the bilateral popliteal areas. The lesions have sharp, well-defined margins. Several have faintly raised papular margins that give the centers a slightly concave appearance.

Palpation reveals the complete absence of any surface disturbance, such as scaling or erosion. Thus, no KOH prep can be performed to check for fungal elements. Instead, a shave biopsy is performed, the results of which show a sawtooth-patterned lymphocytic infiltrate obliterating the normally smooth undulating dermoepidermal junction.

What’s the diagnosis?

 

 

DISCUSSION
This case effectively demonstrates the principle that, when confronted with round or annular lesions, some providers will rely on the diagnosis of “fungal” even when evidence (eg, failed treatment attempts) suggests otherwise. What that nonresponse should do is signal the need for an expanded differential—that is, a consideration of other diagnostic possibilities. This is a bedrock principle in every medical specialty, not just in dermatology.

In this case, the biopsy results clearly pointed to the correct diagnosis of lichen planus (LP), a common dermatosis well known to present in annular morphology. LP is a benign process, albeit one that is occasionally quite bothersome (eg, itching) and, rarely, widespread. LP’s more typical distribution is on volar wrists, in the sacral areas, and occasionally on genitals, so the inability to make a visual diagnosis in this case is forgivable.

Although LP’s etiology is unfortunately unknown, what is known is how to treat it: with topical steroids when necessary or “tincture of time,” as in this patient’s asymptomatic case. LP typically resolves on its own, and it has no worrisome import or connections to more serious disease.

But as always, the first step to correct diagnosis is to consider letting go of the old diagnosis—fungal infection—which was clearly incorrect given the lack of response to numerous antifungals. The second step is to consider other possibilities, which would include lichen planus, psoriasis, granuloma annulare, tinea versicolor, and necrobiosis. The third step is to perform a biopsy, which would establish the correct diagnosis with certainty and in turn, dictate correct treatment.

TAKE-HOME LEARNING POINTS

  • There is an extensive differential for round or annular skin lesions that includes many nonfungal causes.
  • When antifungals fail to help, consider other diagnostic possibilities.
  • Perform a biopsy when a visual diagnosis is not possible.
  • Lichen planus (LP) is a common benign inflammatory skin condition that can present with annular lesions.

A 45-year-old woman is referred to dermatology for a “fungal infection” that has failed to respond to the following treatments: topical clotrimazole cream, topical miconazole cream, a 30-day course of oral terbinafine (250 mg/d), and a 2-month course of oral griseofulvin (unknown dose). The lesions are completely asymptomatic but quite worrisome to the patient since they manifested 6 months ago.

She has consulted at least 6 different providers—none of whom was a dermatologist but all of whom were certain of the diagnosis and thus felt no need to refer the patient. However, the passage of time and trail of ineffective treatments finally prompts the (albeit reluctant) decision to send the patient to dermatology.

On questioning, she denies any serious health problems, such as diabetes or immunosuppression. She has had no contact with any animals or children.

A Faux Fungal Affliction

EXAMINATION
The lesions in question total 6; all are uniformly purplish brown, round, and macular, and they range from 5 mm to more than 3 cm. Most are located on the bilateral popliteal areas. The lesions have sharp, well-defined margins. Several have faintly raised papular margins that give the centers a slightly concave appearance.

Palpation reveals the complete absence of any surface disturbance, such as scaling or erosion. Thus, no KOH prep can be performed to check for fungal elements. Instead, a shave biopsy is performed, the results of which show a sawtooth-patterned lymphocytic infiltrate obliterating the normally smooth undulating dermoepidermal junction.

What’s the diagnosis?

 

 

DISCUSSION
This case effectively demonstrates the principle that, when confronted with round or annular lesions, some providers will rely on the diagnosis of “fungal” even when evidence (eg, failed treatment attempts) suggests otherwise. What that nonresponse should do is signal the need for an expanded differential—that is, a consideration of other diagnostic possibilities. This is a bedrock principle in every medical specialty, not just in dermatology.

In this case, the biopsy results clearly pointed to the correct diagnosis of lichen planus (LP), a common dermatosis well known to present in annular morphology. LP is a benign process, albeit one that is occasionally quite bothersome (eg, itching) and, rarely, widespread. LP’s more typical distribution is on volar wrists, in the sacral areas, and occasionally on genitals, so the inability to make a visual diagnosis in this case is forgivable.

Although LP’s etiology is unfortunately unknown, what is known is how to treat it: with topical steroids when necessary or “tincture of time,” as in this patient’s asymptomatic case. LP typically resolves on its own, and it has no worrisome import or connections to more serious disease.

But as always, the first step to correct diagnosis is to consider letting go of the old diagnosis—fungal infection—which was clearly incorrect given the lack of response to numerous antifungals. The second step is to consider other possibilities, which would include lichen planus, psoriasis, granuloma annulare, tinea versicolor, and necrobiosis. The third step is to perform a biopsy, which would establish the correct diagnosis with certainty and in turn, dictate correct treatment.

TAKE-HOME LEARNING POINTS

  • There is an extensive differential for round or annular skin lesions that includes many nonfungal causes.
  • When antifungals fail to help, consider other diagnostic possibilities.
  • Perform a biopsy when a visual diagnosis is not possible.
  • Lichen planus (LP) is a common benign inflammatory skin condition that can present with annular lesions.
Publications
Publications
Topics
Article Type
Display Headline
A Faux Fungal Affliction
Display Headline
A Faux Fungal Affliction
Sections
Disallow All Ads
Content Gating
No Gating (article Unlocked/Free)
Alternative CME
Disqus Comments
Default
Gate On Date
Tue, 02/12/2019 - 09:45
Un-Gate On Date
Tue, 02/12/2019 - 09:45
Use ProPublica
CFC Schedule Remove Status
Tue, 02/12/2019 - 09:45
Hide sidebar & use full width
render the right sidebar.

A Lesion Hits Its Growth Spurt

Article Type
Changed
Thu, 01/31/2019 - 00:01
Display Headline
A Lesion Hits Its Growth Spurt

When she was 3 years old, a lesion appeared on this child’s face. It was small and caused little to no concern for several years. The child is now 9, and about a year ago, the lesion began to enlarge, ultimately reaching its present size.

First thought to be a pimple, the lesion was later deemed to be “cystic in nature” by another provider. By that point, however, the lesion was quite prominent—to the extent that it intrudes into the patient’s visual field. Perhaps more significantly for someone her age, it has prompted looks and comments that make her uncomfortable.

Fortunately, the lesion causes no pain or physical discomfort, and no other lesions have manifested. The child’s health is generally excellent.

A Lesion Hits Its Growth Spurt

EXAMINATION
A firm nodule, measuring 1.0 by 0.8 cm, is located on the patient’s left upper nasal sidewall. It stands out on an otherwise pristine face free of other blemishes. The lesion is predominantly red, with faint epidermal disturbance in the center. No punctum is appreciated. The lesion is quite firm on palpation, with just a hint of fluctuance but no tenderness or increased warmth.

Excision is clearly indicated; however, the wait for an appointment with a plastic surgeon is currently weeks to months. So an attempt is made to reduce the prominence of the lesion through incision and drainage, which also offers an opportunity to visualize its contents and possibly confirm a diagnosis. The lesion is opened with a #11 blade, and copious amounts of whitish, grainy material is digitally extruded.

What’s the diagnosis?

 

 

DISCUSSION
The contents are consistent with those of a somewhat unusual lesion, commonly called pilomatricoma. It is also known as calcifying epithelioma of Malherbe and pilomatrixoma.

This type of cyst is derived from the hair matrix and is commonly seen on the face, neck, scalp, and arms of children and young adults. This patient’s lesion was atypical in its prominence and erythema, at odds with the firm bluish intradermal papule or nodule usually seen in these cases. But the unique contents established the diagnosis with considerable certainty.

All that remained was the excision—which, given the patient’s age and the cosmetic concerns, would require above-average surgical skills. Once removed, the sample will be sent for pathologic examination, which should show anucleate squamous cells (“ghost cells”), benign viable squamous cells with a lining consisting of basaloid cells. Calcifications with foreign body giant cells account for the pathognomic white flecks seen in the extruded material.

Pilomatricoma’s cause is debatable, but it appears to involve increased levels of beta catenin caused by mutations of the APC gene. This effectively inhibits apoptosis, leading to focal increases in cell growth.

The differential for this type of lesion includes simple acne cyst (unlikely in such a young child), carbuncle (which would have been quite painful and full of pus), or even squamous cell carcinoma.

TAKE-HOME LEARNING POINTS

  • Pilomatricomas are benign cysts usually seen on the face, neck, scalp, and arms of children and young adults.
  • The typical pilomatricoma (sometimes called calcifying epithelioma of Malherbe) is an intradermal papule or nodule, often displaying a faintly bluish color, that is relatively firm on palpation.
  • The contents of a pilomatricoma usually consist of whitish curds or flecks of material that represent calcified tissue mixed with foreign body giant cells.
  • Pilomatricoma has little or no malignant potential but is often cosmetically significant.
Publications
Topics
Sections

When she was 3 years old, a lesion appeared on this child’s face. It was small and caused little to no concern for several years. The child is now 9, and about a year ago, the lesion began to enlarge, ultimately reaching its present size.

First thought to be a pimple, the lesion was later deemed to be “cystic in nature” by another provider. By that point, however, the lesion was quite prominent—to the extent that it intrudes into the patient’s visual field. Perhaps more significantly for someone her age, it has prompted looks and comments that make her uncomfortable.

Fortunately, the lesion causes no pain or physical discomfort, and no other lesions have manifested. The child’s health is generally excellent.

A Lesion Hits Its Growth Spurt

EXAMINATION
A firm nodule, measuring 1.0 by 0.8 cm, is located on the patient’s left upper nasal sidewall. It stands out on an otherwise pristine face free of other blemishes. The lesion is predominantly red, with faint epidermal disturbance in the center. No punctum is appreciated. The lesion is quite firm on palpation, with just a hint of fluctuance but no tenderness or increased warmth.

Excision is clearly indicated; however, the wait for an appointment with a plastic surgeon is currently weeks to months. So an attempt is made to reduce the prominence of the lesion through incision and drainage, which also offers an opportunity to visualize its contents and possibly confirm a diagnosis. The lesion is opened with a #11 blade, and copious amounts of whitish, grainy material is digitally extruded.

What’s the diagnosis?

 

 

DISCUSSION
The contents are consistent with those of a somewhat unusual lesion, commonly called pilomatricoma. It is also known as calcifying epithelioma of Malherbe and pilomatrixoma.

This type of cyst is derived from the hair matrix and is commonly seen on the face, neck, scalp, and arms of children and young adults. This patient’s lesion was atypical in its prominence and erythema, at odds with the firm bluish intradermal papule or nodule usually seen in these cases. But the unique contents established the diagnosis with considerable certainty.

All that remained was the excision—which, given the patient’s age and the cosmetic concerns, would require above-average surgical skills. Once removed, the sample will be sent for pathologic examination, which should show anucleate squamous cells (“ghost cells”), benign viable squamous cells with a lining consisting of basaloid cells. Calcifications with foreign body giant cells account for the pathognomic white flecks seen in the extruded material.

Pilomatricoma’s cause is debatable, but it appears to involve increased levels of beta catenin caused by mutations of the APC gene. This effectively inhibits apoptosis, leading to focal increases in cell growth.

The differential for this type of lesion includes simple acne cyst (unlikely in such a young child), carbuncle (which would have been quite painful and full of pus), or even squamous cell carcinoma.

TAKE-HOME LEARNING POINTS

  • Pilomatricomas are benign cysts usually seen on the face, neck, scalp, and arms of children and young adults.
  • The typical pilomatricoma (sometimes called calcifying epithelioma of Malherbe) is an intradermal papule or nodule, often displaying a faintly bluish color, that is relatively firm on palpation.
  • The contents of a pilomatricoma usually consist of whitish curds or flecks of material that represent calcified tissue mixed with foreign body giant cells.
  • Pilomatricoma has little or no malignant potential but is often cosmetically significant.

When she was 3 years old, a lesion appeared on this child’s face. It was small and caused little to no concern for several years. The child is now 9, and about a year ago, the lesion began to enlarge, ultimately reaching its present size.

First thought to be a pimple, the lesion was later deemed to be “cystic in nature” by another provider. By that point, however, the lesion was quite prominent—to the extent that it intrudes into the patient’s visual field. Perhaps more significantly for someone her age, it has prompted looks and comments that make her uncomfortable.

Fortunately, the lesion causes no pain or physical discomfort, and no other lesions have manifested. The child’s health is generally excellent.

A Lesion Hits Its Growth Spurt

EXAMINATION
A firm nodule, measuring 1.0 by 0.8 cm, is located on the patient’s left upper nasal sidewall. It stands out on an otherwise pristine face free of other blemishes. The lesion is predominantly red, with faint epidermal disturbance in the center. No punctum is appreciated. The lesion is quite firm on palpation, with just a hint of fluctuance but no tenderness or increased warmth.

Excision is clearly indicated; however, the wait for an appointment with a plastic surgeon is currently weeks to months. So an attempt is made to reduce the prominence of the lesion through incision and drainage, which also offers an opportunity to visualize its contents and possibly confirm a diagnosis. The lesion is opened with a #11 blade, and copious amounts of whitish, grainy material is digitally extruded.

What’s the diagnosis?

 

 

DISCUSSION
The contents are consistent with those of a somewhat unusual lesion, commonly called pilomatricoma. It is also known as calcifying epithelioma of Malherbe and pilomatrixoma.

This type of cyst is derived from the hair matrix and is commonly seen on the face, neck, scalp, and arms of children and young adults. This patient’s lesion was atypical in its prominence and erythema, at odds with the firm bluish intradermal papule or nodule usually seen in these cases. But the unique contents established the diagnosis with considerable certainty.

All that remained was the excision—which, given the patient’s age and the cosmetic concerns, would require above-average surgical skills. Once removed, the sample will be sent for pathologic examination, which should show anucleate squamous cells (“ghost cells”), benign viable squamous cells with a lining consisting of basaloid cells. Calcifications with foreign body giant cells account for the pathognomic white flecks seen in the extruded material.

Pilomatricoma’s cause is debatable, but it appears to involve increased levels of beta catenin caused by mutations of the APC gene. This effectively inhibits apoptosis, leading to focal increases in cell growth.

The differential for this type of lesion includes simple acne cyst (unlikely in such a young child), carbuncle (which would have been quite painful and full of pus), or even squamous cell carcinoma.

TAKE-HOME LEARNING POINTS

  • Pilomatricomas are benign cysts usually seen on the face, neck, scalp, and arms of children and young adults.
  • The typical pilomatricoma (sometimes called calcifying epithelioma of Malherbe) is an intradermal papule or nodule, often displaying a faintly bluish color, that is relatively firm on palpation.
  • The contents of a pilomatricoma usually consist of whitish curds or flecks of material that represent calcified tissue mixed with foreign body giant cells.
  • Pilomatricoma has little or no malignant potential but is often cosmetically significant.
Publications
Publications
Topics
Article Type
Display Headline
A Lesion Hits Its Growth Spurt
Display Headline
A Lesion Hits Its Growth Spurt
Sections
Disallow All Ads
Content Gating
No Gating (article Unlocked/Free)
Alternative CME
Disqus Comments
Default
Gate On Date
Tue, 01/29/2019 - 18:30
Un-Gate On Date
Tue, 01/29/2019 - 18:30
Use ProPublica
CFC Schedule Remove Status
Tue, 01/29/2019 - 18:30

Biopsy? What Biopsy?

Article Type
Changed
Thu, 01/17/2019 - 00:01
Display Headline
Biopsy? What Biopsy?

This 80-year-old man has been complaining to health care providers about the asymptomatic lesion on his right forearm for at least 5 years—“maybe more,” he says. During that time, he has been given a variety of diagnoses, mostly “infection” of some sort, along with prescriptions for oral antibiotics (cephalexin, trimethoprim/sulfa), topical antibiotics (mupirocin, triple-antibiotic cream), and germicidal washes (povidone and others). None of these treatment attempts has achieved results.

Prior to now, there has been no referral to dermatology, nor has any provider suggested biopsy of the lesion. The patient has a history of skin cancer (basal cell or squamous cell carcinomas) on his face, back, and arms. These followed a lifetime of sun exposure due to his work in construction and roofing.

Biopsy? What Biopsy?

EXAMINATION
The lesion in question is located on the dorsal aspect of his mid right forearm. Measuring almost 4 cm in aggregate, the lesion is composed of 2 adjacent bright red half-moon friable nodules in a circular configuration. There is almost no erythema or edema in or around the lesion, which is neither warm nor tender to touch.

There are no palpable nodes in the adjacent epitrochlear or axillary nodal locations.

The surrounding skin of this arm, as well as that of the opposite arm, is quite thin, discolored, and scaly and is covered with stellate scars. Examination of all other sun-exposed areas reveals similar changes but no other notable lesions.

What’s the diagnosis?

 

 

DISCUSSION
It would be obvious to most readers that a biopsy is in order, given the likelihood that this lesion is either a basal cell or squamous cell carcinoma—an impression bolstered by the contextual findings of advanced sun damage and the history of skin cancer. And yet, this patient finds himself with a nonhealing friable lesion that is at least 5 years old.

Repeated misdiagnosis as infection is actually a common occurrence. One suspects that at least some of the providers making this mistake followed the patient’s lead: Many affected patients assume such lesions represent infection and therefore demand treatment appropriate for that diagnosis.

As a rule, however, bacterial infections don’t last years, especially in the absence of pain and swelling. Yes, there are nonbacterial infections (eg, those caused by acid-fast bacilli) that can be just as, if not more, indolent, but they too are apt to be painful and swollen.

Biopsy, the obvious missing step in this entire saga, would rule out any of these odd things (including melanoma, which has been known to present in this atypical morphologic manner). Other items in the differential include pyogenic granuloma and metastatic cancer (eg, breast, colon, lung).

In this case, biopsy confirmed the lesion was (as expected) a basal cell carcinoma. The lesion was excised in 2 stages over a 2.5-month period. And the patient was reminded that he will remain at extremely high risk for additional sun-caused skin cancers for the rest of his life.

TAKE-HOME LEARNING POINTS

  • Nonhealing lesions should be assumed to be skin cancer until proven otherwise, especially in sun-damaged individuals.
  • Correct treatment is dictated by correct diagnosis; in this case, a biopsy was clearly indicated.
  • Besides the more common cancers (eg, basal cell and squamous cell carcinoma), there were numerous other items in the differential, including melanoma and even metastatic cancer.
  • Only rarely do skin infections smolder for years without pain or surrounding erythema.
Publications
Topics
Sections

This 80-year-old man has been complaining to health care providers about the asymptomatic lesion on his right forearm for at least 5 years—“maybe more,” he says. During that time, he has been given a variety of diagnoses, mostly “infection” of some sort, along with prescriptions for oral antibiotics (cephalexin, trimethoprim/sulfa), topical antibiotics (mupirocin, triple-antibiotic cream), and germicidal washes (povidone and others). None of these treatment attempts has achieved results.

Prior to now, there has been no referral to dermatology, nor has any provider suggested biopsy of the lesion. The patient has a history of skin cancer (basal cell or squamous cell carcinomas) on his face, back, and arms. These followed a lifetime of sun exposure due to his work in construction and roofing.

Biopsy? What Biopsy?

EXAMINATION
The lesion in question is located on the dorsal aspect of his mid right forearm. Measuring almost 4 cm in aggregate, the lesion is composed of 2 adjacent bright red half-moon friable nodules in a circular configuration. There is almost no erythema or edema in or around the lesion, which is neither warm nor tender to touch.

There are no palpable nodes in the adjacent epitrochlear or axillary nodal locations.

The surrounding skin of this arm, as well as that of the opposite arm, is quite thin, discolored, and scaly and is covered with stellate scars. Examination of all other sun-exposed areas reveals similar changes but no other notable lesions.

What’s the diagnosis?

 

 

DISCUSSION
It would be obvious to most readers that a biopsy is in order, given the likelihood that this lesion is either a basal cell or squamous cell carcinoma—an impression bolstered by the contextual findings of advanced sun damage and the history of skin cancer. And yet, this patient finds himself with a nonhealing friable lesion that is at least 5 years old.

Repeated misdiagnosis as infection is actually a common occurrence. One suspects that at least some of the providers making this mistake followed the patient’s lead: Many affected patients assume such lesions represent infection and therefore demand treatment appropriate for that diagnosis.

As a rule, however, bacterial infections don’t last years, especially in the absence of pain and swelling. Yes, there are nonbacterial infections (eg, those caused by acid-fast bacilli) that can be just as, if not more, indolent, but they too are apt to be painful and swollen.

Biopsy, the obvious missing step in this entire saga, would rule out any of these odd things (including melanoma, which has been known to present in this atypical morphologic manner). Other items in the differential include pyogenic granuloma and metastatic cancer (eg, breast, colon, lung).

In this case, biopsy confirmed the lesion was (as expected) a basal cell carcinoma. The lesion was excised in 2 stages over a 2.5-month period. And the patient was reminded that he will remain at extremely high risk for additional sun-caused skin cancers for the rest of his life.

TAKE-HOME LEARNING POINTS

  • Nonhealing lesions should be assumed to be skin cancer until proven otherwise, especially in sun-damaged individuals.
  • Correct treatment is dictated by correct diagnosis; in this case, a biopsy was clearly indicated.
  • Besides the more common cancers (eg, basal cell and squamous cell carcinoma), there were numerous other items in the differential, including melanoma and even metastatic cancer.
  • Only rarely do skin infections smolder for years without pain or surrounding erythema.

This 80-year-old man has been complaining to health care providers about the asymptomatic lesion on his right forearm for at least 5 years—“maybe more,” he says. During that time, he has been given a variety of diagnoses, mostly “infection” of some sort, along with prescriptions for oral antibiotics (cephalexin, trimethoprim/sulfa), topical antibiotics (mupirocin, triple-antibiotic cream), and germicidal washes (povidone and others). None of these treatment attempts has achieved results.

Prior to now, there has been no referral to dermatology, nor has any provider suggested biopsy of the lesion. The patient has a history of skin cancer (basal cell or squamous cell carcinomas) on his face, back, and arms. These followed a lifetime of sun exposure due to his work in construction and roofing.

Biopsy? What Biopsy?

EXAMINATION
The lesion in question is located on the dorsal aspect of his mid right forearm. Measuring almost 4 cm in aggregate, the lesion is composed of 2 adjacent bright red half-moon friable nodules in a circular configuration. There is almost no erythema or edema in or around the lesion, which is neither warm nor tender to touch.

There are no palpable nodes in the adjacent epitrochlear or axillary nodal locations.

The surrounding skin of this arm, as well as that of the opposite arm, is quite thin, discolored, and scaly and is covered with stellate scars. Examination of all other sun-exposed areas reveals similar changes but no other notable lesions.

What’s the diagnosis?

 

 

DISCUSSION
It would be obvious to most readers that a biopsy is in order, given the likelihood that this lesion is either a basal cell or squamous cell carcinoma—an impression bolstered by the contextual findings of advanced sun damage and the history of skin cancer. And yet, this patient finds himself with a nonhealing friable lesion that is at least 5 years old.

Repeated misdiagnosis as infection is actually a common occurrence. One suspects that at least some of the providers making this mistake followed the patient’s lead: Many affected patients assume such lesions represent infection and therefore demand treatment appropriate for that diagnosis.

As a rule, however, bacterial infections don’t last years, especially in the absence of pain and swelling. Yes, there are nonbacterial infections (eg, those caused by acid-fast bacilli) that can be just as, if not more, indolent, but they too are apt to be painful and swollen.

Biopsy, the obvious missing step in this entire saga, would rule out any of these odd things (including melanoma, which has been known to present in this atypical morphologic manner). Other items in the differential include pyogenic granuloma and metastatic cancer (eg, breast, colon, lung).

In this case, biopsy confirmed the lesion was (as expected) a basal cell carcinoma. The lesion was excised in 2 stages over a 2.5-month period. And the patient was reminded that he will remain at extremely high risk for additional sun-caused skin cancers for the rest of his life.

TAKE-HOME LEARNING POINTS

  • Nonhealing lesions should be assumed to be skin cancer until proven otherwise, especially in sun-damaged individuals.
  • Correct treatment is dictated by correct diagnosis; in this case, a biopsy was clearly indicated.
  • Besides the more common cancers (eg, basal cell and squamous cell carcinoma), there were numerous other items in the differential, including melanoma and even metastatic cancer.
  • Only rarely do skin infections smolder for years without pain or surrounding erythema.
Publications
Publications
Topics
Article Type
Display Headline
Biopsy? What Biopsy?
Display Headline
Biopsy? What Biopsy?
Sections
Disallow All Ads
Content Gating
No Gating (article Unlocked/Free)
Alternative CME
Disqus Comments
Default
Gate On Date
Tue, 01/15/2019 - 10:15
Un-Gate On Date
Tue, 01/15/2019 - 10:15
Use ProPublica
CFC Schedule Remove Status
Tue, 01/15/2019 - 10:15

All the Little Lesions in a Row

Article Type
Changed
Thu, 01/03/2019 - 10:11
Display Headline
All the Little Lesions in a Row

A 10-year-old boy has had a lesion on his left foot for almost a year. It has not responded to either topical antifungal cream (econazole, applied twice daily for weeks) or, subsequently, topical corticosteroid cream (mometazone, also applied twice daily). Frustrated by the lack of resolution, his mother brings him for evaluation.

The condition began with faint linear scaling, the area of which became gradually wider and longer. The child reports no associated symptoms, and the mother denies seeing her son manipulate, rub, or scratch the affected skin.

Aside from mild atopy—in the form of seasonal allergies and asthma—the boy is healthy.

All the Little Lesions in a Row

EXAMINATION
The child is well developed, well nourished, and in no distress. He gladly permits examination of the lesion, which is located on the dorsum of the left foot, running from the lower leg to just proximal to the toes. The linear strip of skin measures 2 cm at its widest point. The lesion is tan and uniformly scaly; it exhibits no overt signs of inflammation or increased warmth or tenderness on palpation.

Examination reveals no other such lesions, or indeed any abnormalities. The adjacent toenails do not appear to be involved.

What is the diagnosis?

 

 

DISCUSSION
This child has a common condition called lichen striatus in modern times, but also known as linear lichenoid dermatitis, or (in older texts) Blaschko linear acquired inflammatory skin eruption. It has nothing to do with fungal infection.

This case illustrates a fairly typical presentation, but—as with most dermatologic conditions—there are many variants. For example, lichen striatus can present as a linear collection of scaly skin running the entire length of the leg (often beginning on the buttocks) and can even affect the toenails at its distal terminus. Although the line is usually solitary, lichen striatus can affect multiple locations simultaneously. Likewise, the lesions can run in an uninterrupted line, or stop and start at various points.

Skin type can affect the appearance of the lesions: on children with darker skin, lichen striatus usually appears lighter and on fair-skinned children, darker. The condition is more common in girls than boys (3:1) and occurs most often in those ages 5 to 15. The arms are another typical location, but it can even affect the face in rare instances. There is some support for atopy as a predisposing factor—but since almost 20% of all children are atopic, this is debatable.

Lichen striatus received its historical name because it follows Blaschko’s lines—named for Alfred Blaschko, a German dermatologist who first described the condition in 1901. These bizarre curving lines are now known to follow recognized patterns of embryonic cell migration that have nothing to do with neural, lymphatic, or vascular patterns as one might otherwise imagine. Several other skin conditions involve so-called blaschkoid features, including inflammatory linear verruciform nevi and some forms of epidermal nevi.

LS is not dangerous in any way, though it does cause considerable consternation among parents of affected children. Luckily, it causes few if any symptoms and is self-limited. A few children will complain of mild itching, for which class 4 or 5 topical steroids can be used. Within a year or two at most, the condition will resolve—albeit with occasional postinflammatory hyperpigmentation in those with darker skin.

TAKE-HOME LEARNING POINTS

  • Lichen striatus is a common condition affecting children ages 5 to 15 who develop a linear, papulosquamous eruption that favors arms and leg (but can rarely involve the face).
  • Not infrequently, the condition can cause dystrophy of the nails at the terminus of the lesions.
  • The lesions follow Blaschko’s lines, which are thought to represent patterns of embryonic cell migration.
  • The condition is seldom symptomatic, is self-limited, and rarely leaves permanent signs of damage.
Publications
Topics
Sections

A 10-year-old boy has had a lesion on his left foot for almost a year. It has not responded to either topical antifungal cream (econazole, applied twice daily for weeks) or, subsequently, topical corticosteroid cream (mometazone, also applied twice daily). Frustrated by the lack of resolution, his mother brings him for evaluation.

The condition began with faint linear scaling, the area of which became gradually wider and longer. The child reports no associated symptoms, and the mother denies seeing her son manipulate, rub, or scratch the affected skin.

Aside from mild atopy—in the form of seasonal allergies and asthma—the boy is healthy.

All the Little Lesions in a Row

EXAMINATION
The child is well developed, well nourished, and in no distress. He gladly permits examination of the lesion, which is located on the dorsum of the left foot, running from the lower leg to just proximal to the toes. The linear strip of skin measures 2 cm at its widest point. The lesion is tan and uniformly scaly; it exhibits no overt signs of inflammation or increased warmth or tenderness on palpation.

Examination reveals no other such lesions, or indeed any abnormalities. The adjacent toenails do not appear to be involved.

What is the diagnosis?

 

 

DISCUSSION
This child has a common condition called lichen striatus in modern times, but also known as linear lichenoid dermatitis, or (in older texts) Blaschko linear acquired inflammatory skin eruption. It has nothing to do with fungal infection.

This case illustrates a fairly typical presentation, but—as with most dermatologic conditions—there are many variants. For example, lichen striatus can present as a linear collection of scaly skin running the entire length of the leg (often beginning on the buttocks) and can even affect the toenails at its distal terminus. Although the line is usually solitary, lichen striatus can affect multiple locations simultaneously. Likewise, the lesions can run in an uninterrupted line, or stop and start at various points.

Skin type can affect the appearance of the lesions: on children with darker skin, lichen striatus usually appears lighter and on fair-skinned children, darker. The condition is more common in girls than boys (3:1) and occurs most often in those ages 5 to 15. The arms are another typical location, but it can even affect the face in rare instances. There is some support for atopy as a predisposing factor—but since almost 20% of all children are atopic, this is debatable.

Lichen striatus received its historical name because it follows Blaschko’s lines—named for Alfred Blaschko, a German dermatologist who first described the condition in 1901. These bizarre curving lines are now known to follow recognized patterns of embryonic cell migration that have nothing to do with neural, lymphatic, or vascular patterns as one might otherwise imagine. Several other skin conditions involve so-called blaschkoid features, including inflammatory linear verruciform nevi and some forms of epidermal nevi.

LS is not dangerous in any way, though it does cause considerable consternation among parents of affected children. Luckily, it causes few if any symptoms and is self-limited. A few children will complain of mild itching, for which class 4 or 5 topical steroids can be used. Within a year or two at most, the condition will resolve—albeit with occasional postinflammatory hyperpigmentation in those with darker skin.

TAKE-HOME LEARNING POINTS

  • Lichen striatus is a common condition affecting children ages 5 to 15 who develop a linear, papulosquamous eruption that favors arms and leg (but can rarely involve the face).
  • Not infrequently, the condition can cause dystrophy of the nails at the terminus of the lesions.
  • The lesions follow Blaschko’s lines, which are thought to represent patterns of embryonic cell migration.
  • The condition is seldom symptomatic, is self-limited, and rarely leaves permanent signs of damage.

A 10-year-old boy has had a lesion on his left foot for almost a year. It has not responded to either topical antifungal cream (econazole, applied twice daily for weeks) or, subsequently, topical corticosteroid cream (mometazone, also applied twice daily). Frustrated by the lack of resolution, his mother brings him for evaluation.

The condition began with faint linear scaling, the area of which became gradually wider and longer. The child reports no associated symptoms, and the mother denies seeing her son manipulate, rub, or scratch the affected skin.

Aside from mild atopy—in the form of seasonal allergies and asthma—the boy is healthy.

All the Little Lesions in a Row

EXAMINATION
The child is well developed, well nourished, and in no distress. He gladly permits examination of the lesion, which is located on the dorsum of the left foot, running from the lower leg to just proximal to the toes. The linear strip of skin measures 2 cm at its widest point. The lesion is tan and uniformly scaly; it exhibits no overt signs of inflammation or increased warmth or tenderness on palpation.

Examination reveals no other such lesions, or indeed any abnormalities. The adjacent toenails do not appear to be involved.

What is the diagnosis?

 

 

DISCUSSION
This child has a common condition called lichen striatus in modern times, but also known as linear lichenoid dermatitis, or (in older texts) Blaschko linear acquired inflammatory skin eruption. It has nothing to do with fungal infection.

This case illustrates a fairly typical presentation, but—as with most dermatologic conditions—there are many variants. For example, lichen striatus can present as a linear collection of scaly skin running the entire length of the leg (often beginning on the buttocks) and can even affect the toenails at its distal terminus. Although the line is usually solitary, lichen striatus can affect multiple locations simultaneously. Likewise, the lesions can run in an uninterrupted line, or stop and start at various points.

Skin type can affect the appearance of the lesions: on children with darker skin, lichen striatus usually appears lighter and on fair-skinned children, darker. The condition is more common in girls than boys (3:1) and occurs most often in those ages 5 to 15. The arms are another typical location, but it can even affect the face in rare instances. There is some support for atopy as a predisposing factor—but since almost 20% of all children are atopic, this is debatable.

Lichen striatus received its historical name because it follows Blaschko’s lines—named for Alfred Blaschko, a German dermatologist who first described the condition in 1901. These bizarre curving lines are now known to follow recognized patterns of embryonic cell migration that have nothing to do with neural, lymphatic, or vascular patterns as one might otherwise imagine. Several other skin conditions involve so-called blaschkoid features, including inflammatory linear verruciform nevi and some forms of epidermal nevi.

LS is not dangerous in any way, though it does cause considerable consternation among parents of affected children. Luckily, it causes few if any symptoms and is self-limited. A few children will complain of mild itching, for which class 4 or 5 topical steroids can be used. Within a year or two at most, the condition will resolve—albeit with occasional postinflammatory hyperpigmentation in those with darker skin.

TAKE-HOME LEARNING POINTS

  • Lichen striatus is a common condition affecting children ages 5 to 15 who develop a linear, papulosquamous eruption that favors arms and leg (but can rarely involve the face).
  • Not infrequently, the condition can cause dystrophy of the nails at the terminus of the lesions.
  • The lesions follow Blaschko’s lines, which are thought to represent patterns of embryonic cell migration.
  • The condition is seldom symptomatic, is self-limited, and rarely leaves permanent signs of damage.
Publications
Publications
Topics
Article Type
Display Headline
All the Little Lesions in a Row
Display Headline
All the Little Lesions in a Row
Sections
Disallow All Ads
Content Gating
No Gating (article Unlocked/Free)
Alternative CME
Disqus Comments
Default
Gate On Date
Thu, 01/03/2019 - 09:30
Un-Gate On Date
Thu, 01/03/2019 - 09:30
Use ProPublica
CFC Schedule Remove Status
Thu, 01/03/2019 - 09:30

How I Became a Derm Guru (And How You Can, Too)

Article Type
Changed
Wed, 12/19/2018 - 10:25
Display Headline
How I Became a Derm Guru (And How You Can, Too)

Many years ago, when I was still in primary care (internal medicine), I thought I knew a bit about the practice of medicine. I was totally comfortable in the hospital (in those days, we saw our own patients twice a day in the hospital), including the ER, the OR, even obstetrics. MIs, shootings, stab wounds, renal failure—I would never say I had mastered them, but I was comfortable with most of what I saw. Deliveries, assisting with C-sections, performing lumbar punctures, performing and interpreting exercise tolerance tests, performing flexible sigmoidoscopies—no problem.

But the one thing that nearly always stopped me in my tracks was … you guessed it: dermatology complaints. Rashes, lesions, or any other skin complaint the least bit out of the ordinary were completely baffling to me. I still remember that feeling after all these years (and I still occasionally experience it!).

I felt like saying to those patients: What in the world would make you think I’d have any idea what that is? But of course, I couldn’t say that, so I’d mumble something, throw some cream at it, then quickly change the subject. Mind you, this was in a setting where a derm referral from us would take 4 to 6 months. And in case you’re wondering, the other providers in my department were as bad at derm as I was.

Long story short, it got to the point that I would scan my schedule every morning, praying I wouldn’t see the word “rash” or “skin.” But, of course, they still came—often just as my hand touched the doorknob to leave: “Oh, by the way, what about this …?” You get the picture. Many of you, if not most, live that picture.

I finally got up the nerve to go to our dermatology department to ask if I could follow one of the docs while he saw patients. Little did I know that practically every provider in the building had already done the same, and had been dismissed with words that essentially meant, “You? A mere PA? You can’t get there from here. Just send ’em to us.”

For a short time, I bought that line—but in the meantime, my patients were not getting the care they needed. So, driven in part by anger at the notion that a mere PA was simply unable to learn dermatology, I bought a decent textbook, Fitzpatrick’s Color Atlas of Dermatology, and started reading it. I also started collecting all the derm articles I could find in the journals, and read about those cases.

I won’t bore you with the grimy details, but what I did differently was work at learning derm (what a concept!). I started going to derm conferences, bought a good camera and started taking pictures with it, and continued to buy books (this was in the pre-computer days of the ’80s) and actually read them.

Continue to: And a funny thing happened...

 

 

And a funny thing happened: The more I read, the more diagnoses I recognized on my patients. My colleagues and the clinic schedulers took note of this and began sending me their problem cases. Even the derm department, beleaguered as usual by huge backlogs of patients, started sending patients to me. By 1985, even though I was in the internal medicine department, I had transitioned to doing derm fulltime. And that’s what I’ve been doing since.

Around 1992, I discovered that I was one of 6 dermatology PAs in this country. Last time I checked, our numbers were approaching 4,000. So, yes, derm is indeed difficult, but rocket science it isn’t.

Being the pedantic sort that I am, and finding that whole experience so enlightening, I resolved to make it my mission to foster the use of PAs in dermatology—part of which involves the education of those PAs, by means of taking students but also by writing articles (several hundred at last count) and lecturing at conferences and at PA programs. Nearing retirement, I only practice two days a week, but I write and publish at least 5 clinical articles a month, all of which are based on real cases: my cases, using my photos, doing new research on each case. This keeps my knowledge fresh and my 75-year-old mind sharp, helps ward off burnout, and, most importantly, saves lives while reducing patient discomfort.

What follows are 10 dermatology pearls that I have gleaned along the way. My apologies to my former students and attendees at my lectures who’ve heard all this before:

1 If the treatment for your diagnosis isn’t working, consider another diagnosis. Here’s an example (Figure 1): A man in his 50s was sent to dermatology for psoriasis that wasn’t responding to a biologic. Was it really psoriasis? A KOH prep quickly showed it to be tinea corporis, which cleared completely with a month’s worth of oral terbinafine (250 mg qid).

Dermatophytosis misdiagnosed as psoriasis

Continue to: #2...

 

 

2 The correct diagnosis dictates correct treatment. This may sound obvious, but in primary care, the emphasis is often on “let’s try this” or “let’s try that,” an understandable approach to a symptomatic patient with an uncertain diagnosis. But by the time he finally gets to dermatology, the patient has tried a whole bag full of prescription and OTC products given for numerous, totally different diagnoses. A better approach might be to expedite an urgent referral to dermatology, when possible.

3 Cutaneous fungal infections (ie, dermatophytosis) are vastly overdiagnosed, especially by novices. If you truly suspect it, ask about a potential source; one doesn’t acquire a fungal infection out of thin air. It must come from a person, animal, or occasionally, the soil. It also helps if the victim has been rendered susceptible by the injudicious use of steroids. Better yet, find the fungus with a microscopic examination (KOH prep) or culture. Finally, remember, not everything round and scaly is fungal (see Figure 2).

Granuloma annulare

4 Remember these ancient words of wisdom regarding skin complaints: (a) A diagnosis is seldom made if not entertained, (b) you won’t entertain it if you’ve never heard of it, (c) you will not see it if you’re not looking for it, and (d) even if you did see it, you would not “see” it because you’re not looking for it. Dermatology is far deeper and wider than most imagine it to be. The trick is to expose yourself to as many different diagnoses as possible, by reading and attending lectures, ahead of the possible sighting. Figures 3 and 4 offer examples of common conditions that are seldom recognized outside dermatology.

5 Skin cancer can present as a rash. Examples abound, such as mammary and extramammary Paget’s disease (Figure 5), mycosis fungoides, metastatic breast cancer (Figure 6), and superficial basal cell carcinoma. A biopsy is usually required to diagnose these, but you wouldn’t think to do that if you’d never heard of the condition.

Paget’s disease, type 2 and Metastatic breast cancer

6 Melanoma doesn’t typically arise from a mole or other pre-existing lesion. Far more often, it arises “de novo,” out of nothing. So, in general, we’re not worried about “moles” (nevi) unless there’s a history of change (see Figure 7).

Melanoma

Continue to: #7...

 

 

7 When looking for skin cancer, pay as much attention to the owner as to the lesion. The most common skin cancers—basal cell and squamous cell carcinoma—usually occur on sun-damaged, fair-skinned, blue-eyed older patients. Though there are certainly exceptions to this paradigm, it pays to be generally suspicious of any odd lesion seen on these patients (Figure 8).

Dermatoheliosis

8 It’s practically impossible to overstate the role of atopy when evaluating pediatric skin complaints. These children—20% of all newborns!—are born with thin, dry, sensitive, overreactive skin that is prone to eczema and urticaria. They will also have a marked tendency to develop seasonal allergies, allergic rhinitis, and asthma. Parents find it difficult to accept the genetic basis for atopic dermatitis (Figure 9), preferring instead to blame everything on laundry detergent or food. Education (of oneself first!) is the key.

Atopic dermatitis

9 “Infections” are not always what they seem. These pearls may help: a) There are many causes of localized erythema, swelling, and tenderness that have nothing to do with infection. Examples include relapsing polychondritis (Figure 10) and cutaneous lupus (Figure 11). b) Skin cancer, such as basal cell carcinoma (see Figure 12), is often mistaken for infection. c) Some infections have less conventional or obvious sources; those caused by atypical mycobacteria (see Figure 13), leishmaniasis (see Figure 14), and sporothrix (see Figure 15), are good examples of this phenomenon. Consider thinking outside the box.

Relapsing polychondritis; Cutaneous lupus; Perinasal basal cell carcinoma

10 Overcome your fear of steroids by educating yourself about their safe use. Glucocorticoids (eg, triamcinolone, prednisone, betamethasone) are extremely useful in treating common derm conditions. We see patients every day who are so frightened of steroids, they won’t even consider using them because some well-meaning medical provider scared them to death. The proper use of these miraculous products could easily be the subject of an entire article. For now, I’ll advise you to read about their safe use in any number of dermatology texts (including online publications).

M. fortuitum infection; Old-world leishmaniasis; Sporothrix

Article PDF
Author and Disclosure Information

Joe R. Monroe, MPAS, PA, practices at Dermatology Associates of Oklahoma in Tulsa. He is also the founder of the Society of Dermatology Physician Assistants.

Issue
Clinician Reviews - 28(12)
Publications
Topics
Page Number
27-30
Sections
Author and Disclosure Information

Joe R. Monroe, MPAS, PA, practices at Dermatology Associates of Oklahoma in Tulsa. He is also the founder of the Society of Dermatology Physician Assistants.

Author and Disclosure Information

Joe R. Monroe, MPAS, PA, practices at Dermatology Associates of Oklahoma in Tulsa. He is also the founder of the Society of Dermatology Physician Assistants.

Article PDF
Article PDF

Many years ago, when I was still in primary care (internal medicine), I thought I knew a bit about the practice of medicine. I was totally comfortable in the hospital (in those days, we saw our own patients twice a day in the hospital), including the ER, the OR, even obstetrics. MIs, shootings, stab wounds, renal failure—I would never say I had mastered them, but I was comfortable with most of what I saw. Deliveries, assisting with C-sections, performing lumbar punctures, performing and interpreting exercise tolerance tests, performing flexible sigmoidoscopies—no problem.

But the one thing that nearly always stopped me in my tracks was … you guessed it: dermatology complaints. Rashes, lesions, or any other skin complaint the least bit out of the ordinary were completely baffling to me. I still remember that feeling after all these years (and I still occasionally experience it!).

I felt like saying to those patients: What in the world would make you think I’d have any idea what that is? But of course, I couldn’t say that, so I’d mumble something, throw some cream at it, then quickly change the subject. Mind you, this was in a setting where a derm referral from us would take 4 to 6 months. And in case you’re wondering, the other providers in my department were as bad at derm as I was.

Long story short, it got to the point that I would scan my schedule every morning, praying I wouldn’t see the word “rash” or “skin.” But, of course, they still came—often just as my hand touched the doorknob to leave: “Oh, by the way, what about this …?” You get the picture. Many of you, if not most, live that picture.

I finally got up the nerve to go to our dermatology department to ask if I could follow one of the docs while he saw patients. Little did I know that practically every provider in the building had already done the same, and had been dismissed with words that essentially meant, “You? A mere PA? You can’t get there from here. Just send ’em to us.”

For a short time, I bought that line—but in the meantime, my patients were not getting the care they needed. So, driven in part by anger at the notion that a mere PA was simply unable to learn dermatology, I bought a decent textbook, Fitzpatrick’s Color Atlas of Dermatology, and started reading it. I also started collecting all the derm articles I could find in the journals, and read about those cases.

I won’t bore you with the grimy details, but what I did differently was work at learning derm (what a concept!). I started going to derm conferences, bought a good camera and started taking pictures with it, and continued to buy books (this was in the pre-computer days of the ’80s) and actually read them.

Continue to: And a funny thing happened...

 

 

And a funny thing happened: The more I read, the more diagnoses I recognized on my patients. My colleagues and the clinic schedulers took note of this and began sending me their problem cases. Even the derm department, beleaguered as usual by huge backlogs of patients, started sending patients to me. By 1985, even though I was in the internal medicine department, I had transitioned to doing derm fulltime. And that’s what I’ve been doing since.

Around 1992, I discovered that I was one of 6 dermatology PAs in this country. Last time I checked, our numbers were approaching 4,000. So, yes, derm is indeed difficult, but rocket science it isn’t.

Being the pedantic sort that I am, and finding that whole experience so enlightening, I resolved to make it my mission to foster the use of PAs in dermatology—part of which involves the education of those PAs, by means of taking students but also by writing articles (several hundred at last count) and lecturing at conferences and at PA programs. Nearing retirement, I only practice two days a week, but I write and publish at least 5 clinical articles a month, all of which are based on real cases: my cases, using my photos, doing new research on each case. This keeps my knowledge fresh and my 75-year-old mind sharp, helps ward off burnout, and, most importantly, saves lives while reducing patient discomfort.

What follows are 10 dermatology pearls that I have gleaned along the way. My apologies to my former students and attendees at my lectures who’ve heard all this before:

1 If the treatment for your diagnosis isn’t working, consider another diagnosis. Here’s an example (Figure 1): A man in his 50s was sent to dermatology for psoriasis that wasn’t responding to a biologic. Was it really psoriasis? A KOH prep quickly showed it to be tinea corporis, which cleared completely with a month’s worth of oral terbinafine (250 mg qid).

Dermatophytosis misdiagnosed as psoriasis

Continue to: #2...

 

 

2 The correct diagnosis dictates correct treatment. This may sound obvious, but in primary care, the emphasis is often on “let’s try this” or “let’s try that,” an understandable approach to a symptomatic patient with an uncertain diagnosis. But by the time he finally gets to dermatology, the patient has tried a whole bag full of prescription and OTC products given for numerous, totally different diagnoses. A better approach might be to expedite an urgent referral to dermatology, when possible.

3 Cutaneous fungal infections (ie, dermatophytosis) are vastly overdiagnosed, especially by novices. If you truly suspect it, ask about a potential source; one doesn’t acquire a fungal infection out of thin air. It must come from a person, animal, or occasionally, the soil. It also helps if the victim has been rendered susceptible by the injudicious use of steroids. Better yet, find the fungus with a microscopic examination (KOH prep) or culture. Finally, remember, not everything round and scaly is fungal (see Figure 2).

Granuloma annulare

4 Remember these ancient words of wisdom regarding skin complaints: (a) A diagnosis is seldom made if not entertained, (b) you won’t entertain it if you’ve never heard of it, (c) you will not see it if you’re not looking for it, and (d) even if you did see it, you would not “see” it because you’re not looking for it. Dermatology is far deeper and wider than most imagine it to be. The trick is to expose yourself to as many different diagnoses as possible, by reading and attending lectures, ahead of the possible sighting. Figures 3 and 4 offer examples of common conditions that are seldom recognized outside dermatology.

5 Skin cancer can present as a rash. Examples abound, such as mammary and extramammary Paget’s disease (Figure 5), mycosis fungoides, metastatic breast cancer (Figure 6), and superficial basal cell carcinoma. A biopsy is usually required to diagnose these, but you wouldn’t think to do that if you’d never heard of the condition.

Paget’s disease, type 2 and Metastatic breast cancer

6 Melanoma doesn’t typically arise from a mole or other pre-existing lesion. Far more often, it arises “de novo,” out of nothing. So, in general, we’re not worried about “moles” (nevi) unless there’s a history of change (see Figure 7).

Melanoma

Continue to: #7...

 

 

7 When looking for skin cancer, pay as much attention to the owner as to the lesion. The most common skin cancers—basal cell and squamous cell carcinoma—usually occur on sun-damaged, fair-skinned, blue-eyed older patients. Though there are certainly exceptions to this paradigm, it pays to be generally suspicious of any odd lesion seen on these patients (Figure 8).

Dermatoheliosis

8 It’s practically impossible to overstate the role of atopy when evaluating pediatric skin complaints. These children—20% of all newborns!—are born with thin, dry, sensitive, overreactive skin that is prone to eczema and urticaria. They will also have a marked tendency to develop seasonal allergies, allergic rhinitis, and asthma. Parents find it difficult to accept the genetic basis for atopic dermatitis (Figure 9), preferring instead to blame everything on laundry detergent or food. Education (of oneself first!) is the key.

Atopic dermatitis

9 “Infections” are not always what they seem. These pearls may help: a) There are many causes of localized erythema, swelling, and tenderness that have nothing to do with infection. Examples include relapsing polychondritis (Figure 10) and cutaneous lupus (Figure 11). b) Skin cancer, such as basal cell carcinoma (see Figure 12), is often mistaken for infection. c) Some infections have less conventional or obvious sources; those caused by atypical mycobacteria (see Figure 13), leishmaniasis (see Figure 14), and sporothrix (see Figure 15), are good examples of this phenomenon. Consider thinking outside the box.

Relapsing polychondritis; Cutaneous lupus; Perinasal basal cell carcinoma

10 Overcome your fear of steroids by educating yourself about their safe use. Glucocorticoids (eg, triamcinolone, prednisone, betamethasone) are extremely useful in treating common derm conditions. We see patients every day who are so frightened of steroids, they won’t even consider using them because some well-meaning medical provider scared them to death. The proper use of these miraculous products could easily be the subject of an entire article. For now, I’ll advise you to read about their safe use in any number of dermatology texts (including online publications).

M. fortuitum infection; Old-world leishmaniasis; Sporothrix

Many years ago, when I was still in primary care (internal medicine), I thought I knew a bit about the practice of medicine. I was totally comfortable in the hospital (in those days, we saw our own patients twice a day in the hospital), including the ER, the OR, even obstetrics. MIs, shootings, stab wounds, renal failure—I would never say I had mastered them, but I was comfortable with most of what I saw. Deliveries, assisting with C-sections, performing lumbar punctures, performing and interpreting exercise tolerance tests, performing flexible sigmoidoscopies—no problem.

But the one thing that nearly always stopped me in my tracks was … you guessed it: dermatology complaints. Rashes, lesions, or any other skin complaint the least bit out of the ordinary were completely baffling to me. I still remember that feeling after all these years (and I still occasionally experience it!).

I felt like saying to those patients: What in the world would make you think I’d have any idea what that is? But of course, I couldn’t say that, so I’d mumble something, throw some cream at it, then quickly change the subject. Mind you, this was in a setting where a derm referral from us would take 4 to 6 months. And in case you’re wondering, the other providers in my department were as bad at derm as I was.

Long story short, it got to the point that I would scan my schedule every morning, praying I wouldn’t see the word “rash” or “skin.” But, of course, they still came—often just as my hand touched the doorknob to leave: “Oh, by the way, what about this …?” You get the picture. Many of you, if not most, live that picture.

I finally got up the nerve to go to our dermatology department to ask if I could follow one of the docs while he saw patients. Little did I know that practically every provider in the building had already done the same, and had been dismissed with words that essentially meant, “You? A mere PA? You can’t get there from here. Just send ’em to us.”

For a short time, I bought that line—but in the meantime, my patients were not getting the care they needed. So, driven in part by anger at the notion that a mere PA was simply unable to learn dermatology, I bought a decent textbook, Fitzpatrick’s Color Atlas of Dermatology, and started reading it. I also started collecting all the derm articles I could find in the journals, and read about those cases.

I won’t bore you with the grimy details, but what I did differently was work at learning derm (what a concept!). I started going to derm conferences, bought a good camera and started taking pictures with it, and continued to buy books (this was in the pre-computer days of the ’80s) and actually read them.

Continue to: And a funny thing happened...

 

 

And a funny thing happened: The more I read, the more diagnoses I recognized on my patients. My colleagues and the clinic schedulers took note of this and began sending me their problem cases. Even the derm department, beleaguered as usual by huge backlogs of patients, started sending patients to me. By 1985, even though I was in the internal medicine department, I had transitioned to doing derm fulltime. And that’s what I’ve been doing since.

Around 1992, I discovered that I was one of 6 dermatology PAs in this country. Last time I checked, our numbers were approaching 4,000. So, yes, derm is indeed difficult, but rocket science it isn’t.

Being the pedantic sort that I am, and finding that whole experience so enlightening, I resolved to make it my mission to foster the use of PAs in dermatology—part of which involves the education of those PAs, by means of taking students but also by writing articles (several hundred at last count) and lecturing at conferences and at PA programs. Nearing retirement, I only practice two days a week, but I write and publish at least 5 clinical articles a month, all of which are based on real cases: my cases, using my photos, doing new research on each case. This keeps my knowledge fresh and my 75-year-old mind sharp, helps ward off burnout, and, most importantly, saves lives while reducing patient discomfort.

What follows are 10 dermatology pearls that I have gleaned along the way. My apologies to my former students and attendees at my lectures who’ve heard all this before:

1 If the treatment for your diagnosis isn’t working, consider another diagnosis. Here’s an example (Figure 1): A man in his 50s was sent to dermatology for psoriasis that wasn’t responding to a biologic. Was it really psoriasis? A KOH prep quickly showed it to be tinea corporis, which cleared completely with a month’s worth of oral terbinafine (250 mg qid).

Dermatophytosis misdiagnosed as psoriasis

Continue to: #2...

 

 

2 The correct diagnosis dictates correct treatment. This may sound obvious, but in primary care, the emphasis is often on “let’s try this” or “let’s try that,” an understandable approach to a symptomatic patient with an uncertain diagnosis. But by the time he finally gets to dermatology, the patient has tried a whole bag full of prescription and OTC products given for numerous, totally different diagnoses. A better approach might be to expedite an urgent referral to dermatology, when possible.

3 Cutaneous fungal infections (ie, dermatophytosis) are vastly overdiagnosed, especially by novices. If you truly suspect it, ask about a potential source; one doesn’t acquire a fungal infection out of thin air. It must come from a person, animal, or occasionally, the soil. It also helps if the victim has been rendered susceptible by the injudicious use of steroids. Better yet, find the fungus with a microscopic examination (KOH prep) or culture. Finally, remember, not everything round and scaly is fungal (see Figure 2).

Granuloma annulare

4 Remember these ancient words of wisdom regarding skin complaints: (a) A diagnosis is seldom made if not entertained, (b) you won’t entertain it if you’ve never heard of it, (c) you will not see it if you’re not looking for it, and (d) even if you did see it, you would not “see” it because you’re not looking for it. Dermatology is far deeper and wider than most imagine it to be. The trick is to expose yourself to as many different diagnoses as possible, by reading and attending lectures, ahead of the possible sighting. Figures 3 and 4 offer examples of common conditions that are seldom recognized outside dermatology.

5 Skin cancer can present as a rash. Examples abound, such as mammary and extramammary Paget’s disease (Figure 5), mycosis fungoides, metastatic breast cancer (Figure 6), and superficial basal cell carcinoma. A biopsy is usually required to diagnose these, but you wouldn’t think to do that if you’d never heard of the condition.

Paget’s disease, type 2 and Metastatic breast cancer

6 Melanoma doesn’t typically arise from a mole or other pre-existing lesion. Far more often, it arises “de novo,” out of nothing. So, in general, we’re not worried about “moles” (nevi) unless there’s a history of change (see Figure 7).

Melanoma

Continue to: #7...

 

 

7 When looking for skin cancer, pay as much attention to the owner as to the lesion. The most common skin cancers—basal cell and squamous cell carcinoma—usually occur on sun-damaged, fair-skinned, blue-eyed older patients. Though there are certainly exceptions to this paradigm, it pays to be generally suspicious of any odd lesion seen on these patients (Figure 8).

Dermatoheliosis

8 It’s practically impossible to overstate the role of atopy when evaluating pediatric skin complaints. These children—20% of all newborns!—are born with thin, dry, sensitive, overreactive skin that is prone to eczema and urticaria. They will also have a marked tendency to develop seasonal allergies, allergic rhinitis, and asthma. Parents find it difficult to accept the genetic basis for atopic dermatitis (Figure 9), preferring instead to blame everything on laundry detergent or food. Education (of oneself first!) is the key.

Atopic dermatitis

9 “Infections” are not always what they seem. These pearls may help: a) There are many causes of localized erythema, swelling, and tenderness that have nothing to do with infection. Examples include relapsing polychondritis (Figure 10) and cutaneous lupus (Figure 11). b) Skin cancer, such as basal cell carcinoma (see Figure 12), is often mistaken for infection. c) Some infections have less conventional or obvious sources; those caused by atypical mycobacteria (see Figure 13), leishmaniasis (see Figure 14), and sporothrix (see Figure 15), are good examples of this phenomenon. Consider thinking outside the box.

Relapsing polychondritis; Cutaneous lupus; Perinasal basal cell carcinoma

10 Overcome your fear of steroids by educating yourself about their safe use. Glucocorticoids (eg, triamcinolone, prednisone, betamethasone) are extremely useful in treating common derm conditions. We see patients every day who are so frightened of steroids, they won’t even consider using them because some well-meaning medical provider scared them to death. The proper use of these miraculous products could easily be the subject of an entire article. For now, I’ll advise you to read about their safe use in any number of dermatology texts (including online publications).

M. fortuitum infection; Old-world leishmaniasis; Sporothrix

Issue
Clinician Reviews - 28(12)
Issue
Clinician Reviews - 28(12)
Page Number
27-30
Page Number
27-30
Publications
Publications
Topics
Article Type
Display Headline
How I Became a Derm Guru (And How You Can, Too)
Display Headline
How I Became a Derm Guru (And How You Can, Too)
Sections
Disallow All Ads
Content Gating
No Gating (article Unlocked/Free)
Alternative CME
Disqus Comments
Default
Use ProPublica
Article PDF Media